Sie sind auf Seite 1von 56

USMLE Step 1

Sample Test Questions


A Joint Program of the Federation of State Medical Boards of the United States, Inc.,
and the National Board of Medical Examiners

This booklet updated April 2014.


Copyright 2014 by the Federation of State Medical Boards of the United States, Inc. (FSMB), and the
National Board of Medical Examiners (NBME). All rights reserved. Printed in the United States of
America. The United States Medical Licensing Examination (USMLE) is a joint program of the FSMB and
the NBME.

CONTENTS

USMLE Step 1 Test Question Formats .. 3


Introduction to USMLE Step 1 Sample Test Questions 4
Normal Laboratory Values .. 5
USMLE Step 1 Sample Test Questions.. 7
Answer Form for USMLE Step 1 Sample Test Questions. 42
Answer Key for USMLE Step 1 Sample Test Questions 43

USMLE Step 1 Test Question Formats


Strategies
The following are strategies for answering one-best-answer items:

Read each question carefully. It is important to understand what is being asked.

Try to generate an answer and then look for it in the option list.

Alternatively, read each option carefully, eliminating those that are clearly incorrect.

Of the remaining options, select the one that is most correct.

If unsure about an answer, it is better to guess since unanswered questions are automatically counted as wrong
answers.

Single-Item Questions
A single patient-centered vignette is associated with one question followed by four or more response options. The
response options are lettered (ie, A, B, C, D, E). A portion of the questions involves interpretation of graphic or pictorial
materials. You are required to select the best answer to the question. Other options may be partially correct, but there is
only ONE BEST answer. This is the traditional, most frequently used multiple-choice question format on the examination.
Example Item
A 32-year-old woman with type 1 diabetes mellitus has had progressive renal failure over the past 2 years. She has not yet
started dialysis. Examination shows no abnormalities. Her hemoglobin concentration is 9 g/dL, hematocrit is 28%, and
mean corpuscular volume is 94 m3. A blood smear shows normochromic, normocytic cells. Which of the following is the
most likely cause?
(A)
(B)
(C)
(D)
(E)
(F)

Acute blood loss


Chronic lymphocytic leukemia
Erythrocyte enzyme deficiency
Erythropoietin deficiency
Immunohemolysis
Microangiopathic hemolysis

(G)
(H)
(I)
(J)

Polycythemia vera
Sickle cell disease
Sideroblastic anemia
-Thalassemia trait

(Answer: D)
Sequential Item Sets
A single patient-centered vignette may be associated with two or three consecutive questions about the information
presented. Each question is associated with the initial patient vignette but is testing a different point. You are required to
select the ONE BEST answer to each question. Questions are designed to be answered in sequential order. You must click
Proceed to Next Item to view the next item in the set; once you click on this button, you will not be able to add or
change an answer to the displayed (previous) item.
NOTE: Some item types that appear on the Step 1 examination are NOT depicted in the sample items provided in this
booklet: sequential item sets and items with multimedia features (such as audio). In addition, when additional item
formats are added to the exam, notice will be provided at the USMLE Web site: www.usmle.org. You must monitor the
Web site to stay informed about the types of items that occur in the exam, and must practice with the downloadable
sample test items available on the USMLE Web site to be fully prepared for the examination.

Introduction to USMLE Step 1 Sample Test Questions


The following pages include 138 sample test questions. Most of these questions are the same as those you install on your
computer from the USMLE Web site. Please note that reviewing the sample questions as they appear on pages 7-41 is not
a substitute for practicing with the test software. You should download and run the Step 1 tutorial and practice test items
that are provided on the USMLE website well before your test date. The sample materials available at the USMLE Web
site include additional items and item formats that do not appear in this booklet, such as items with associated audio or
video findings and sequential item sets. You should become familiar with all item formats that will be used in the actual
examination.
Although the sample questions exemplify content on the examination, they may not reflect the content coverage on
individual examinations. In the actual examination, questions will be presented in random order; they will not be grouped
according to specific content. The questions will be presented one at a time in a format designed for easy on-screen
reading, including use of exhibit buttons for the Normal Laboratory Values Table (included here on pages 56) and some
pictorials. Photographs, charts, and x-rays in this booklet are not of the same quality as the pictorials used in the actual
examination. In addition, you will be able to adjust the brightness and contrast of pictorials on the computer screen.
To take the following sample test questions as they would be timed in the actual examination, you should allow a
maximum of one hour for each 46-item block, for a total of three hours. Please be aware that most examinees perceive the
time pressure to be greater during an actual examination. An answer form for recording answers is provided on page 42.
An answer key is provided on page 43. In the actual examination, answers will be selected on the screen; no answer form
will be provided.

LABORATORY VALUES
* Included in the Biochemical Profile (SMA-12)
REFERENCE RANGE
SI REFERENCE INTERVALS
BLOOD, PLASMA, SERUM
* Alanine aminotransferase (ALT), serum ................. 8-20 U/L ................................................... 8-20 U/L
Amylase, serum ....................................................... 25-125 U/L ................................................ 25-125 U/L
* Aspartate aminotransferase (AST), serum .............. 8-20 U/L .................................................... 8-20 U/L
Bilirubin, serum (adult) Total // Direct ................... 0.1-1.0 mg/dL // 0.0-0.3 mg/dL ................ 2-17 mol/L // 0-5 mol/L
* Calcium, serum (Ca2+) ............................................8.4-10.2 mg/dL .......................................... 2.1-2.8 mmol/L
* Cholesterol, serum .................................................. Rec:<200 mg/dL ...................................... <5.2 mmol/L
Cortisol, serum ........................................................ 0800 h: 5-23 g/dL // 1600 h: 3-15 g/dL 138-635 nmol/L // 82-413 nmol/L
2000 h: < 50% of 0800 h ........................... Fraction of 0800 h: < 0.50
Creatine kinase, serum ............................................Male: 25-90 U/L ....................................... 25-90 U/L
Female: 10-70 U/L ................................... 10-70 U/L
* Creatinine, serum .................................................... 0.6-1.2 mg/dL ........................................... 53-106 mol/L
Electrolytes, serum
Sodium (Na+) ........................................................ 136-145 mEq/L ......................................... 136-145 mmol/L
* Potassium (K+) ...................................................... 3.5-5.0 mEq/L ........................................... 3.5-5.0 mmol/L
Chloride (Cl) ........................................................ 95-105 mEq/L .......................................... 95-105 mmol/L
Bicarbonate (HCO3) ............................................. 22-28 mEq/L ............................................ 22-28 mmol/L
Magnesium (Mg2+) ................................................ 1.5-2.0 mEq/L ........................................... 0.75-1.0 mmol/L
Estriol, total, serum (in pregnancy)
24-28 wks // 32-36 wks .........................................30-170 ng/mL // 60-280 ng/mL ................ 104-590 nmol/L // 208-970 nmol/L
28-32 wks // 36-40 wks .........................................40-220 ng/mL // 80-350 ng/mL ................ 140-760 nmol/L // 280-1210 nmol/L
Ferritin, serum ......................................................... Male: 15-200 ng/mL ................................ 15-200 g/L
Female: 12-150 ng/mL ............................. 12-150 g/L
Follicle-stimulating hormone, serum/plasma .........Male: 4-25 mIU/mL ................................. 4-25 U/L
Female: premenopause 4-30 mIU/mL ...... 4-30 U/L
midcycle peak 10-90 mIU/mL ............... 10-90 U/L
postmenopause 40-250 mIU/mL ........... 40-250 U/L
Gases, arterial blood (room air)
pH .........................................................................7.35-7.45 .................................................. [H+] 36-44 nmol/L
PCO2 ......................................................................33-45 mm Hg ............................................ 4.4-5.9 kPa
PO2 ........................................................................75-105 mm Hg .......................................... 10.0-14.0 kPa
* Glucose, serum ........................................................ Fasting: 70-110 mg/dL ............................. 3.8-6.1 mmol/L
2-h postprandial: < 120 mg/dL ................ < 6.6 mmol/L
Growth hormone - arginine stimulation .................. Fasting: < 5 ng/mL ................................... < 5 g/L
provocative stimuli: > 7 ng/mL ............. > 7 g/L
Immunoglobulins, serum
IgA .......................................................................76-390 mg/dL ............................................ 0.76-3.90 g/L
IgE ........................................................................0-380 IU/mL ............................................ 0-380 kIU/L
IgG .......................................................................650-1500 mg/dL ....................................... 6.5-15 g/L
IgM .......................................................................40-345 mg/dL ........................................... 0.4-3.45 g/L
Iron .........................................................................50-170 g/dL ............................................ 9-30 mol/L
Lactate dehydrogenase, serum ................................ 45-90 U/L .................................................. 45-90 U/L
Luteinizing hormone, serum/plasma ...................... Male: 6-23 mIU/mL ................................. 6-23 U/L
Female: follicular phase 5-30 mIU/mL .... 5-30 U/L
midcycle 75-150 mIU/mL ...................... 75-150 U/L
postmenopause 30-200 mIU/mL ........... 30-200 U/L
Osmolality, serum ................................................... 275-295 mOsmol/kg H2O ......................... 275-295 mOsmol/kg H2O
Parathyroid hormone, serum, N-terminal ............... 230-630 pg/mL ......................................... 230-630 ng/L
* Phosphatase (alkaline), serum (p-NPP at 30C) .....20-70 U/L ................................................. 20-70 U/L
* Phosphorus (inorganic), serum ................................ 3.0-4.5 mg/dL ........................................... 1.0-1.5 mmol/L
Prolactin, serum (hPRL) .........................................< 20 ng/mL ............................................... < 20 g/L
* Proteins, serum
Total (recumbent) ................................................. 6.0-7.8 g/dL .............................................. 60-78 g/L
Albumin ................................................................ 3.5-5.5 g/dL ............................................... 35-55 g/L
Globulin ............................................................... 2.3-3.5 g/dL ............................................... 23-35 g/L
Thyroid-stimulating hormone, serum or plasma .....0.5-5.0 U/mL .......................................... 0.5-5.0 mU/L
Thyroidal iodine (123I) uptake ..................................8%-30% of administered dose/24 h .......... 0.08-0.30/24 h
Thyroxine (T4), serum ............................................. 5-12 g/dL ................................................ 64-155 nmol/L
Triglycerides, serum................................................ 35-160 mg/dL ............................................ 0.4-1.81 mmol/L
Triiodothyronine (T3), serum (RIA) ....................... 115-190 ng/dL .......................................... 1.8-2.9 nmol/L
Triiodothyronine (T3) resin uptake .......................... 25%-35% .................................................. 0.25-0.35
* Urea nitrogen, serum .............................................. 7-18 mg/dL ............................................... 1.2-3.0 mmol/L
* Uric acid, serum ...................................................... 3.0-8.2 mg/dL ........................................... 0.18-0.48 mmol/L
5

LABORATORY VALUES (continued from previous page)


REFERENCE RANGE
SI REFERENCE INTERVALS
BODY MASS INDEX (BMI)
Body mass index ...................................................... Adult: 19-25 kg/m2
CEREBROSPINAL FLUID
Cell count ................................................................. 0-5/mm3 ............................................................ 0-5 x 106/L
Chloride ................................................................... 118-132 mEq/L ................................................ 118-132 mmol/L
Gamma globulin ....................................................... 3%-12% total proteins ...................................... 0.03-0.12
Glucose ................................................................... 40-70 mg/dL .................................................... 2.2-3.9 mmol/L
Pressure ................................................................... 70-180 mm H2O .............................................. 70-180 mm H2O
Proteins, total .......................................................... <40 mg/dL ...................................................... <0.40 g/L
HEMATOLOGIC
Bleeding time (template) ......................................... 2-7 minutes ....................................................... 2-7 minutes
Erythrocyte count ..................................................... Male: 4.3-5.9 million/mm3 ............................... 4.3-5.9 x 1012/L
Female: 3.5-5.5 million/mm3 ............................ 3.5-5.5 x 1012/L
Erythrocyte sedimentation rate (Westergren)........... Male: 0-15 mm/h ............................................. 0-15 mm/h
Female: 0-20 mm/h .......................................... 0-20 mm/h
Hematocrit ............................................................... Male: 41%-53% ............................................... 0.41-0.53
Female: 36%-46% ............................................ 0.36-0.46
Hemoglobin A1c ....................................................... < 6% ................................................................. < 0.06
Hemoglobin, blood................................................... Male: 13.5-17.5 g/dL ....................................... 2.09-2.71 mmol/L
Female: 12.0-16.0 g/dL .................................... 1.86-2.48 mmol/L
Hemoglobin, plasma ................................................ 1-4 mg/dL ......................................................... 0.16-0.62 mmol/L
Leukocyte count and differential
Leukocyte count ..................................................... 4500-11,000/mm3 ............................................. 4.5-11.0 x 109/L
Segmented neutrophils ......................................... 54%-62% ......................................................... 0.54-0.62
Bands.................................................................... 3%-5% ............................................................. 0.03-0.05
Eosinophils .......................................................... 1%-3% ............................................................. 0.01-0.03
Basophils .............................................................. 0%-0.75% ......................................................... 0-0.0075
Lymphocytes ....................................................... 25%-33% .......................................................... 0.25-0.33
Monocytes ........................................................... 3%-7% ............................................................. 0.03-0.07
Mean corpuscular hemoglobin ................................. 25.4-34.6 pg/cell .............................................. 0.39-0.54 fmol/cell
Mean corpuscular hemoglobin concentration ......... 31%-36% Hb/cell ............................................ 4.81-5.58 mmol Hb/L
Mean corpuscular volume ....................................... 80-100 m3 ....................................................... 80-100 fL
Partial thromboplastin time (activated) ................... 25-40 seconds ................................................... 25-40 seconds
Platelet count ............................................................ 150,000-400,000/mm3 ...................................... 150-400 x 109/L
Prothrombin time ..................................................... 11-15 seconds ................................................... 11-15 seconds
Reticulocyte count.................................................... 0.5%-1.5% ........................................................ 0.005-0.015
Thrombin time ......................................................... <2 seconds deviation from control .................. <2 seconds deviation from control
Volume
Plasma ................................................................... Male: 25-43 mL/kg........................................... 0.025-0.043 L/kg
Female: 28-45 mL/kg ....................................... 0.028-0.045 L/kg
Red cell .................................................................. Male: 20-36 mL/kg .......................................... 0.020-0.036 L/kg
Female: 19-31 mL/kg ...................................... 0.019-0.031 L/kg
SWEAT
Chloride.................................................................... 0-35 mmol/L .................................................... 0-35 mmol/L
URINE
Calcium ................................................................... 100-300 mg/24 h .............................................. 2.5-7.5 mmol/24 h
Chloride.................................................................... Varies with intake ............................................. Varies with intake
Creatinine clearance ................................................. Male: 97-137 mL/min
Female: 88-128 mL/min
Estriol, total (in pregnancy)
30 wks .................................................................... 6-18 mg/24 h .................................................... 21-62 mol/24 h
35 wks .................................................................... 9-28 mg/24 h .................................................... 31-97 mol/24 h
40 wks .................................................................... 13-42 mg/24 h .................................................. 45-146 mol/24 h
17-Hydroxycorticosteroids ...................................... Male: 3.0-10.0 mg/24 h .................................... 8.2-27.6 mol/24 h
Female: 2.0-8.0 mg/24 h................................... 5.5-22.0 mol/24 h
17-Ketosteroids, total ............................................... Male: 8-20 mg/24 h .......................................... 28-70 mol/24 h
Female: 6-15 mg/24 h....................................... 21-52 mol/24 h
Osmolality ............................................................... 50-1400 mOsmol/kg H2O
Oxalate ..................................................................... 8-40 g/mL ...................................................... 90-445 mol/L
Potassium ................................................................ Varies with diet ................................................ Varies with diet
Proteins, total .......................................................... <150 mg/24 h .................................................. <0.15 g/24 h
Sodium .................................................................... Varies with diet ................................................ Varies with diet
Uric acid ................................................................... Varies with diet ................................................ Varies with diet
6

USMLE STEP1 SAMPLE TEST QUESTIONS


BLOCK 1, ITEMS 1-46
1.

A 25-year-old woman is brought to the


emergency department 1 hour after she fainted.
She has had mild intermittent vaginal bleeding,
sometimes associated with lower abdominal
pain, during the past 3 days. She has had severe
cramping pain in the right lower abdomen for
12 hours. She has not had a menstrual period for
3 months; previously, menses occurred at regular
28-day intervals. Abdominal examination shows
mild tenderness to palpation in the right lower
quadrant. Bimanual pelvic examination shows a
tender walnut-sized mass in the right
parametrium. Which of the following is the most
likely diagnosis?
(A)
(B)
(C)
(D)
(E)
(F)

3.

(A) Adenine phosphoribosyltransferase


deficiency
(B) Hypoxanthine-guanine
phosphoribosyltransferase
deficiency
(C) Increased cellular turnover of nucleic
acids
(D) Increased conversion of hypoxanthine
to inosine monophosphate
(E) Phosphoribosylpyrophosphate
synthetase deficiency

Appendicitis
Cancer of the ovary
Ectopic pregnancy
Endometriosis
Ovarian cyst
Placenta previa
4.

2.

A 12-year-old girl with a 1-year history of


systemic lupus erythematosus is brought to the
physician for a routine follow-up examination.
During the past year she has done well with the
exception of occasional mild frontal headaches,
fatigue, and arthralgias; the results of regular
laboratory evaluations have been stable. She has
missed 20 days of school in the past 4 months.
Her parents express concern that sending her to
school might cause her to be exposed to children
with contagious diseases that might exacerbate
their daughter's condition. The girl tells the
physician that she misses seeing her many
friends at school. Passive motion of the elbows,
wrists, and knees produces mild discomfort.
Physical examination shows no other
abnormalities. Which of the following is the
most likely cause of this patient's excessive
school absences?
(A)
(B)
(C)
(D)
(E)

A 4-year-old boy has delayed motor


development and choreoathetosis. He had normal
development at birth. He chews his fingers and
lips, which has resulted in tissue loss. He has
arthritis. Serum and urine uric acid
concentrations are increased. Which of the
following abnormalities is the most likely cause
of these findings?

A 42-year-old woman comes to the physician for


a routine examination. She says that she has felt
well except for occasional episodes of
constipation, abdominal discomfort, and mild
fatigue. She was treated for a renal calculus 10
years ago and was told she had a "lazy
gallbladder." Her pulse is 82/min, and blood
pressure is 150/80 mm Hg. Physical examination
shows no other abnormalities. Laboratory studies
show:
Erythrocyte count
Serum
K+
Cl
Ca2+
Phosphorus
Alkaline phosphatase

3 million/mm3
4.5 mEq/L
107 mEq/L
12 mg/dL
2.2 mg/dL
95 U/L

The most likely cause of this patient's condition


is a small, well-defined nodule in which of the
following locations?

Lupus arthritis
Lupus cerebritis
Malingering
School phobia
Vulnerable child syndrome

(A)
(B)
(C)
(D)
(E)
(F)

Adrenal gland
Anterior pituitary gland
Gallbladder
Kidney
Parathyroid gland
Thymus

7.

5.

6.

(A)
(B)
(C)
(D)
(E)

A 4-year-old girl has the sudden onset of


abdominal pain and vomiting. She has a mass in
the right lower quadrant and hyperactive bowel
sounds. A segment of resected bowel is shown in
the photograph. Which of the following is the
most likely diagnosis?
(A)
(B)
(C)
(D)
(E)

8.

Appendicitis
Intussusception
Meckel diverticulum
Necrotizing enterocolitis
Strangulated hernia

A 12-year-old girl with sickle cell disease has


pain in her right arm. An x-ray of the right upper
extremity shows bony lesions consistent with
osteomyelitis. Which of the following is the most
likely causal organism?
(A)
(B)
(C)
(D)
(E)
(F)
(G)

A 3-year-old boy is brought to the physician


because of fever, headache, and sores on his
back and left shoulder for 1 day. His temperature
is 37.8C (100F). Physical examination shows
vesicles over the back and left shoulder as in the
photograph shown. Treatment with aspirin is
CONTRAINDICATED in this patient because of
an epidemiologic association with a syndrome
that includes which of the following adverse
effects?
Bronchoconstriction
Disseminated intravascular coagulation
Gastric irritation
Hepatomegaly
Immunosuppression

A 62-year-old woman comes to the physician


because of low back pain for 1 week. Menopause
occurred 10 years ago. Physical examination
shows localized tenderness over the lumbar spine
after movement. X-rays of the spine show a
compression fracture of L1-2. A DEXA scan
shows decreased bone mineral density. Serum
calcium and phosphorus concentrations and
serum alkaline phosphatase activity are within
the reference ranges. A bisphosphonate drug is
prescribed. The expected beneficial effect of this
drug is most likely due to which of the following
actions?
(A) Decreased insulin-like growth factor-1
concentration
(B) Decreased osteoclast activity
(C) Decreased osteoprotegerin production
(D) Increased 1,25dihydroxycholecalciferol
concentration
(E) Increased osteoblast activity
(F) Increased receptor activator of NF-B
ligand (RANKL) production

Clostridium septicum
Enterococcus faecalis
Listeria monocytogenes
Proteus mirabilis
Pseudomonas aeruginosa
Salmonella enteritidis
Serratia marcescens

9.

Hospital discharge of a 75-year-old man is delayed due to unavailability of a bed in a nursing home. He is bedridden
and unable to attend to his personal needs. During a 3-day period, his pulse increases from 82/min to 125/min, and
blood pressure decreases from 124/72 mm Hg to 100/55 mm Hg. Laboratory values include:

Hemoglobin
Serum
Urea nitrogen
Glucose
Na+
Creatinine

Day 1
16.4 g/dL

Day 3
18.4 g/dL

18 mg/dL
100 mg/dL
135 mEq/L
1.1 mg/dL

56 mg/dL
89 mg/dL
151 mEq/L
1.2 mg/dL

Which of the following is the most likely diagnosis?


(A)
(B)
(C)
(D)
(E)

10.

Acute renal failure


Dehydration
Diabetic ketoacidosis
Gastrointestinal hemorrhage
Syndrome of inappropriate ADH (vasopressin)

A 76-year-old man with a history of prostatic


hypertrophy has the recent onset of increased
difficulty urinating. Symptoms began shortly
after he started taking a nasal decongestant orally
for cold symptoms. Which of the following types
of receptors is most likely to be involved in these
adverse effects?

12.

1-Adrenergic
2-Adrenergic
Ganglionic nicotinic
Nicotinic receptor at the neuromuscular
junction
(E) Serotoninergic
(A)
(B)
(C)
(D)

11.

(A) Significant findings can be reported


with greater confidence
(B) The study will have more power
(C) There is a decreased likelihood of a
Type II error
(D) There is an increased likelihood of
statistically significant findings
(E) There is an increased likelihood of a
Type I error

A 15-year-old girl is brought to the physician


because of a 3-week history of excessive thirst
and voiding excessive amounts of urine. She
shows no signs of kidney damage, and she is not
taking any medications. Physical examination
shows no abnormalities. She undergoes an 8hour water deprivation test. She is also given
5 units of ADH (vasopressin), subcutaneously.
Under both conditions, she continues to produce
large volumes of dilute urine. Her symptoms are
most likely due to a relative lack of which of the
following proteins from the apical membranes of
collecting duct epithelial cells?
(A)
(B)
(C)
(D)
(E)

A placebo-controlled clinical trial is conducted


to assess whether a new antihypertensive drug is
more effective than standard therapy. A total of
5000 patients with essential hypertension are
enrolled and randomly assigned to one of two
groups: 2500 patients receive the new drug and
2500 patients receive placebo. If the alpha is set
at 0.01 instead of 0.05, which of the following is
the most likely result?

13.

Aquaporin
Epithelial Na+ channel
Na+K+ ATPase
Na+K+2Cl cotransporter
Urea transporter

A sexually active 23-year-old man with multiple


sex partners has dysuria and a yellow urethral
exudate. Gram stain of the exudate shows
numerous neutrophils, many that contain
intracellular gram-negative diplococci. He has
had three similar episodes of urethritis over the
past 2 years. Which of the following properties
of the infecting organism best explains the
reinfection?
(A)
(B)
(C)
(D)
(E)

Antigenic variation
Catalase
Inhibition of B-lymphocyte function
Inhibition of T-lymphocyte function
Polysaccharide capsule

14.

A 68-year-old woman has the sudden onset of weakness in her right arm and leg. She can speak, but her words are
not enunciated clearly. Neurologic examination 6 weeks later shows an extensor plantar reflex on the right. When
she is asked to protrude her tongue, it deviates to the left, and the muscle in the left side of the tongue shows
considerable atrophy. Which of the following labeled areas in the transverse sections of the brain stem is most likely
damaged?

15.

A 33-year-old woman comes to the physician


because of a 2-day history of mild nausea,
increased urinary urgency and frequency, and
constipation. She also has had a 4.5-kg (10-lb)
weight loss during the past 2 weeks and a 3week history of vaginal bleeding. Pelvic
examination shows a nodular cervix with an
irregular, friable posterior lip, and a rock-hard,
irregular, immobile pelvic mass that extends
across the pelvis. Examination of biopsy
specimens from the cervix and anterior wall of
the vagina show well-differentiated keratinizing
squamous cell carcinoma. Which of the
following best describes the pathogenesis of this
patient's disease?

16.

A 22-year-old man develops delusions, flattening


of affect, catatonic behavior, hallucinations, and
aphasia. Which of the following symptoms
would be more likely to improve if this patient
were treated with clozapine rather than with
haloperidol?
(A)
(B)
(C)
(D)
(E)

17.

(A) Inactivation of cellular p53


(B) Insertion of viral promotors adjacent to
cellular growth factor genes
(C) Specialized transduction
(D) Transactivation of cellular growth
factor genes by TAX
(E) Translocation of CMYC to an Ig gene
promoter

After infection with measles virus, a 6-year-old


boy produces antibodies to all eight viral
proteins. The next year he is again exposed to
measles virus. Antibodies to which of the
following viral proteins are most likely to be
protective?
(A)
(B)
(C)
(D)
(E)

10

Affective flattening and aphasia


Affective flattening and hallucinations
Aphasia and delusions
Catatonia and delusions
Hallucinations and catatonia

Hemagglutinin
Matrix
Nonstructural
Nucleocapsid
Polymerase

18.

A 25-year-old woman comes to the physician because of a 10-year history of frequent occurrences of fever blisters.
Physical examination shows perioral vesicles. Microscopic examination of culture of scrapings from three vesicles
shows herpes simplex virus 1. Which of the following patterns in the figure shown was most likely observed when
the viral DNA from the cultures was examined by restriction enzyme analysis on polyacrylamide gels?

19.

A 52-year-old woman comes to the physician


because of a 2-day history of fever and left flank
pain. She has been treated for multiple episodes
of pyelonephritis during the past 3 years. Her
temperature is 37.8C (100.1F). Physical
examination shows left flank tenderness.
Urinalysis shows 1218 WBC/hpf with
occasional lymphocytes and mononuclear cells
with features of macrophages. Cultures of urine
grow 80,000 colonies/mL of Proteus mirabilis.
An x-ray of the abdomen shows a 3-cm mass in
the lower pole of the left kidney. Gross
examination of the mass after it has been
resected shows that it is yellow, 3.2-cm in
diameter, and centrally but not marginally
necrotic. Histologic examination of the mass
shows a predominance of epithelioid cells with
partially clear and granular-to-foamy cytoplasm.
Nuclei are eccentric, normochromic, symmetric,
and without significant pleomorphism. Scattered
lymphocytes and plasma cells are intermixed.
Which of the following is the most likely
diagnosis?

20.

A 35-year-old woman with a bicuspid aortic


valve comes to the physician because of a 1week history of a swollen, painful left knee. She
has had occasional fever and progressive fatigue
during the past 4 months. Her temperature is
38.2C (100.8F). Physical examination shows a
tender left knee with an effusion. A grade 2/6
systolic murmur is heard. Echocardiography
shows a vegetation on the aortic valve. Analysis
of synovial fluid aspirated from the left knee
shows many segmented neutrophils; a Gram
stain shows no organisms, and culture is
negative. Microscopic examination of the urine
shows RBCs and RBC casts, but culture of the
urine grows no organisms. Four blood cultures
grow an -hemolytic streptococcus. Which of the
following is the most likely cause of the joint
and renal disease?
(A) Autoimmune response triggered by a
bacterial antigen
(B) Deposition of antigen-antibody
complexes
(C) IgA nephropathy
(D) Neutrophil response to replicating
bacteria
(E) Type IV (delayed) hypersensitivity to a
bacterial antigen

(A) Acute pyelonephritis


(B) Malacoplakia
(C) Renal cell carcinoma, clear cell type,
intermediate grade
(D) Renal cell carcinoma, granular cell type
(E) Xanthogranulomatous pyelonephritis

11

22.

21.

A 28-year-old man comes to the physician


because of a 1-year history of pain with urination
that has increased in severity during the past
month. He also has had episodes of blood in his
urine during the past 5 years. He lived in subSaharan Africa until he came to the USA
6 months ago for graduate school. His
temperature is 38C (100.4F), pulse is 80/min,
respirations are 16/min, and blood pressure is
110/84 mm Hg. Physical examination shows
suprapubic tenderness. Laboratory studies show:
Hemoglobin
Hematocrit
Leukocyte count
Segmented neutrophils
Bands
Eosinophils
Lymphocytes
Monocytes
Serum
Urea nitrogen
Creatinine
Urine
Blood
RBC
WBC
RBC casts
WBC casts

An otherwise healthy 45-year-old man comes to


the physician because of a 3-week history of
progressive epigastric heartburn and a 4.5-kg
(10-lb) weight loss. The pain tends to be more
severe at night and occurs 1 to 3 hours after
meals during the day. He has had similar
episodes with lesser intensity during the past
year. Abdominal examination shows tenderness
to deep palpation. Test of the stool for occult
blood is positive. Endoscopy shows a bleeding 3cm ulcer in the antrum of the stomach. A
photomicrograph of Steiner silver-stained tissue
(400x) from a biopsy of the gastric mucosa
adjacent to the ulcer is shown. Which of the
following processes is most likely to be
involved?

12.3 g/dL
37%
13,400/mm3
65%
5%
5%
22%
3%
75 mg/dL
3.8 mg/dL
3+
200/hpf
100/hpf
absent
absent

Imaging studies show bilateral hydroureter and


hydronephrosis and foci of calcification in the
region of the bladder. A biopsy specimen of the
bladder shows marked chronic inflammation
with fibrosis and scattered granulomas. Which of
the following best explains the biopsy findings?
(A)
(B)
(C)
(D)
(E)

(A) Elaboration of proteases and urease


with local tissue destruction
(B) Hyperacidity and gastric ulcer
development
(C) Ingestion of preformed toxins in
contaminated well water
(D) Spirochete invasion of gastric cells

12

Exposure to a chemical toxin


Interstitial cystitis
Malacoplakia
Schistosomiasis
Vesicoureteral reflux

23.

A 4-year-old boy is brought to the physician because of slow growth during the past year. He has had recurrent
urinary tract infections since the age of 1 year. He is at the 10th percentile for height and 25th percentile for weight.
Physical examination shows pallor. Laboratory studies show a normochromic, normocytic anemia and increased
serum concentrations of urea nitrogen and creatinine. Urinalysis shows a low specific gravity. Which of the
following sets of additional serum findings is most likely in this patient?

(A)
(B)
(C)
(D)
(E)
(F)

24.

Inorganic Phosphorus

1,25-Dihydroxycholecalciferol

A 38-year-old man who recently immigrated to


the USA comes to the physician because of a 1month history of cough and a 4.5-kg (10-lb)
weight loss. Physical examination shows no
abnormalities. A chest x-ray shows a right upper
lobe infiltrate. One of three sputum samples is
positive for acid-fast bacilli. Treatment with
isoniazid,
rifampin,
ethambutol,
and
pyrazinamide is started. Which of the following
should be added to the medication regimen to
prevent neurologic toxicity in this patient?
(A)
(B)
(C)
(D)
(E)

25.

Calcium

26.

Folic acid
Nicotinic acid
Vitamin B6 (pyridoxine)
Vitamin B12 (cyanocobalamin)
Vitamin C

(A)
(B)
(C)
(D)
(E)

A technician wants to determine whether


cytomegalovirus (CMV) DNA is present in the
blood of a bone marrow transplant recipient.
DNA purified from the leukocytes of the patient
is
reacted
in
a
mixture
containing
oligonucleotides specific for CMV DNA,
thermostable DNA polymerase, and nucleotides.
Repetitive cycles of heating and cooling are
performed, and the reaction product is detected
by gel electrophoresis. The technician most
likely used which of the following laboratory
procedures on this patient's blood?
(A)
(B)
(C)
(D)
(E)

A 12-year-old girl is brought to the emergency


department by her parents because of a 3-day
history of fever and a 12-hour history of
lethargy. Her parents say that she has been
sleeping most of the day and has been
unresponsive when awake. Her temperature is
39.2C (102.6F). Physical examination shows
numerous petechial hemorrhages and nuchal
rigidity. A lumbar puncture yields cloudy
cerebrospinal fluid (CSF) that clots in the
collection tube. Microscopic examination of the
CSF shows numerous segmented neutrophils,
and a Gram stain shows gram-negative
diplococci. Which of the following is the most
likely causal organism?

27.

Haemophilus influenzae
Mycoplasma pneumoniae
Neisseria meningitidis
Salmonella typhi
Streptococcus pneumoniae

Northern blotting
Polymerase chain reaction
Reverse transcription
Southern blotting
Western blotting

A 6-year-old boy from rural Mississippi is


brought to the physician by his mother because
of a 6-month history of lethargy; he also has had
a 4-kg (9-lb) weight loss during this period. The
mother says her son used to be active, often
playing outside without wearing his shoes. The
patient is at the 10th percentile for height and
12th percentile for weight. He appears pale.
Physical examination shows pale oral mucosa.
Laboratory studies show iron deficiency anemia.
Microscopic examination of the stool shows
thin-shelled ova. Which of the following is the
most appropriate pharmacotherapy for this
patient?
(A)
(B)
(C)
(D)
(E)

13

Erythropoietin

Doxycycline
Ivermectin
Mebendazole
Mefloquine
Trimethoprim-sulfamethoxazole

28.

A 17-year-old girl has never had a menstrual


period. Physical examination shows a normal
female body habitus, normal breast development,
and normal appearing external genitalia. She has
no axillary or pubic hair. The patient refuses to
have a pelvic or rectal examination. Which of the
following is the most likely explanation for the
clinical presentation?
(A)
(B)
(C)
(D)
(E)

29.

31.

Androgen insensitivity
Congenital adrenal hyperplasia
Ectodermal dysplasia
A psychiatric disorder
A sex chromosome mosaicism

(A) Autoimmune disorder


(B) Defective ion transport at epithelial
surfaces
(C) Disaccharidase deficiency
(D) Inability to synthesize apolipoprotein B
(E) Villous atrophy of the jejunum

Warfarin is administered to a 56-year-old man


following placement of a prosthetic cardiac
valve. The warfarin dosage is adjusted to
maintain an INR of 2.5. Subsequently,
trimethoprim-sulfamethoxazole therapy is begun
for a recurring urinary tract infection. In addition
to monitoring prothrombin time, which of the
following actions should the physician take to
maintain adequate anticoagulation?

32.

(A) Begin therapy with vitamin K


(B) Increase the dosage of warfarin
(C) Make no alterations in the dosage of
warfarin
(D) Decrease the dosage of warfarin
(E) Stop the warfarin and change to lowdose aspirin

30.

A 50-year-old man has headache, vertigo, and


generalized pruritus. He has the recent onset of
angina pectoris. His hematocrit is 65%,
leukocyte count is 12,000/mm3, and erythrocyte
mass is increased. Erythropoietin concentration
is decreased. Which of the following is the most
likely diagnosis?
(A) Glucose-6-phosphate dehydrogenase
deficiency
(B) Hemochromatosis
(C) Immune thrombocytopenic purpura
(D) Pernicious anemia
(E) Polycythemia vera
(F) Pyruvate kinase deficiency
(G) Secondary polycythemia

A 40-year-old woman comes to the physician


because of a 6-month history of increased facial
hair growth. Her last menstrual period was 4
months ago. She is 165 cm (5 ft 5 in) tall and
weighs 70 kg (154 lb); BMI is 26 kg/m2. Her
pulse is 80/min, and blood pressure is 130/82
mm Hg. Physical examination shows temporal
balding and coarse dark hair on the upper lip and
chin. Pelvic examination shows clitoral
enlargement.
Her
serum
testosterone
concentration is increased. Serum concentrations
of androstenedione, dehydroepiandrosterone, and
urinary 17-ketosteroids are within the reference
ranges. Ultrasonography of the pelvis shows a
12-cm ovarian mass. Which of the following best
describes this mass?
(A)
(B)
(C)
(D)
(E)

A 7-month-old infant is brought to the


physician's office because of poor weight gain
despite large food intake. He has had two
episodes of pneumonia and has frequent bulky
stools. He coughs frequently. X-rays of the lungs
show increased markings and hyperinflation.
Trypsin is absent in a fresh stool sample, and the
fat content is increased. Which of the following
is the most likely cause of this infant's disorder?

33.

A new blood test to detect prostate cancer is


evaluated in 300 male volunteers. A needle
biopsy of the prostate gland is done on all men
with
serum
prostate-specific
antigen
concentrations greater than 5 ng/mL (N<4). One
hundred men undergo biopsy procedures; 90 are
found to have prostate cancer, and five are found
to have chronic prostatitis. Which of the
following is necessary to calculate the sensitivity
of this test?
(A) Incidence of chronic prostatitis in the
general population
(B) Number of men with test results greater
than 5 ng/mL and a normal biopsy
specimen
(C) Prevalence of chronic prostatitis in the
general population
(D) Prostate biopsies of men with test
results equal to or below 5 ng/mL

Granulosa tumor
Ovarian carcinoid
Sertoli-Leydig tumor
Teratoma
Thecoma

14

34.

A 47-year-old man comes to the physician because of a 1-week history of temperatures to 38.3C (101F) and
occasional vomiting. He also has a 1-year history of joint and muscle pain in his calves and a 1-month history of
intermittent, diffuse abdominal pain. His temperature now is 37.2C (99F). Abdominal examination shows mild
diffuse tenderness. There is no ascites. Test of the stool for occult blood is positive. Serum studies show mildly
increased urea nitrogen and creatinine concentrations. Photomicrographs of a biopsy specimen of the mesentery are
shown. Which of the following is the most likely diagnosis?
(A)
(B)
(C)
(D)
(E)
(F)

35.

Angiodysplasia
Mucocutaneous lymph node syndrome (Kawasaki disease)
Polyarteritis nodosa
Takayasu arteritis
Thromboangiitis obliterans
Wegener granulomatosis

A 15-year-old girl comes to the physician


because of a 3-month history of acne. Breast and
pubic hair development began at the age of
12 years. Menarche occurred at the age of
14 years. Physical examination shows scattered
open and closed comedones over the cheeks and
forehead. Breast and pubic hair development are
Tanner stage 5. Which of the following is the
most likely underlying cause of this patient's
acne?

36.

(A) Decreased parasympathetic stimulation


to the sebaceous glands
(B) Increased estrogen stimulation of the
sebaceous glands
(C) Increased responsiveness of the
sebaceous glands to folliclestimulating hormone
(D) Increased sympathetic stimulation to the
sebaceous glands
(E) Stimulation of the sebaceous glands by
androgens

A 6-year-old boy with glioblastoma has a


recurrence of the tumor despite aggressive
treatment. The physician discusses the patient's
prognosis with his parents and recommends
palliative care. The parents ask how they should
talk with their son about his prognosis and
possible death. The physician advises that the
parents should be honest and follow the patient's
lead during the conversation. This patient most
likely has which of the following concepts of
death?
(A)
(B)
(C)
(D)

Being asleep
Being final
Being a long journey
Being a temporary separation from his
parents
(E) No understanding of death

15

37.

A 42-year-old man comes to the physician for a


follow-up examination 1 week after he passed a
renal calculus. X-ray crystallographic analysis of
the calculus showed calcium as the primary
cation. Physical examination today shows no
abnormalities. A 24-hour collection of urine
shows increased calcium excretion. Which of the
following
is
the
most
appropriate
pharmacotherapy?
(A)
(B)
(C)
(D)
(E)

38.

40.

Carbonic anhydrase inhibitor


Na+Cl symport inhibitor
Na+K+2Cl symport inhibitor
Osmotic diuretic
Renal epithelial sodium channel
inhibitor

(A)
(B)
(C)
(D)
(E)

A 35-year-old man who works at a facility


processing highly radioactive substances
accidentally receives a high, whole-body dose of
ionizing
radiation
estimated
to
be
1500 rads (15 gray). He dies 1 week later. At
autopsy, histologic examination of the skin
shows scattered, individual epidermal cells with
shrunken, markedly eosinophilic cytoplasm and
pyknotic, fragmented nuclei. These morphologic
changes most likely indicate which of the
following processes?
(A)
(B)
(C)
(D)
(E)

41.

Apoptosis
Coagulation necrosis
Liquefaction necrosis
Mutagenesis
Tumor initiation

A 52-year-old man comes to the emergency


department because he has had vomiting, nausea,
and abdominal pain for the past 12 hours. He
says he attempted suicide 3 days ago by "taking
everything in the medicine cabinet." He was
stuporous for approximately 12 hours after the
overdose but felt better the following day. At this
time, he has jaundice and pain in the right upper
quadrant. Which of the following drugs is most
likely to have caused the pain, vomiting, and
jaundice?
(A)
(B)
(C)
(D)
(E)

16

Calcitonin
Collagen, type I
1-Hydroxylase
Parathyroid hormone
Vitamin D receptor

A 55-year-old woman with small cell carcinoma


of the lung is admitted to the hospital to undergo
chemotherapy. Six days after treatment is started,
she develops a temperature of 38C (100.4F).
Physical examination shows no other
abnormalities. Laboratory studies show a
leukocyte count of 100/mm3 (5% segmented
neutrophils and 95% lymphocytes). Which of the
following
is
the
most
appropriate
pharmacotherapy to increase this patient's
leukocyte count?
(A)
(B)
(C)
(D)
(E)
(F)

Acetaminophen
Aspirin
Cimetidine
Diphenhydramine
Triazolam

Infraspinatus
Pectoralis
Subscapularis
Supraspinatus
Trapezius

A 2-year-old girl is brought to the emergency


department because of pain in her right forearm
after a fall 1 hour ago. She has a history of
fractures of the left femur and right tibia.
Physical examination shows blue sclerae. There
is tenderness to palpation over the distal right
radius. A mutation in which of the following
genes is the most likely cause of the recurrent
fractures in this patient?
(A)
(B)
(C)
(D)
(E)

42.
39.

A 45-year-old man comes to the physician


because of right shoulder pain that began after he
chopped wood 2 days ago. Examination of the
right upper extremity shows no obvious bone
deformities or point tenderness. The pain is
reproduced when the patient is asked to
externally rotate the shoulder against resistance;
there is no weakness. In addition to the teres
minor, inflammation of which of the following
tendons is most likely in this patient?

Darbepoetin
Dexamethasone
Filgrastim
Interferon alfa
Interleukin-2 (IL-2)
Leucovorin

43.

A 23-year-old man comes to the physician because of a 2-day history of sore throat. Current medications include an
inhaled corticosteroid for asthma. His temperature is 37C (98.6F). A photograph of the tongue is shown. A KOH
preparation of a scraping from one of the plaques shows budding yeast. Which of the following is the most
appropriate pharmacotherapy for this patient?
(A)
(B)
(C)
(D)
(E)
(F)
(G)

44.

Amphotericin B
Caspofungin
Fluconazole
Flucytosine
Itraconazole
Nystatin
Voriconazole

A full-term female newborn is examined shortly


after birth. She appears to be small for
gestational age, and she has excess skin on the
nape of the neck and lymphedema of the hands
and feet. Chromosomal analysis shows some
cells with a normal 46,XY karyotype and some
cells with a 45,X karyotype. Which of the
following mechanisms best explains this
cytogenetic abnormality?
(A)
(B)
(C)
(D)
(E)

45.

Nondisjunction in mitosis
Reciprocal translocation
Robertsonian translocation
Skewed X-inactivation
Uniparental disomy

A 46-year-old woman receives a nonTlymphocyte-depleted, allogeneic bone marrow


transplant from a matched, unrelated donor.
Immunosuppressive therapy with cyclosporine is
started. One month later, she has fever. Cytolytic
destruction of the skin, gastrointestinal tract, and
liver is seen, with associated dermatitis, enteritis,
and hepatitis. Which of the following best
explains these findings?
(A)
(B)
(C)
(D)
(E)

17

C3b deposition
Cytomegalovirus infection
Graft-versus-host disease
Tolerance induction
Type I (immediate) hypersensitivity

46.

A 26-year-old man is brought to the emergency department by ambulance 30 minutes after being shot in the leg. He
is unconscious and appears markedly pale. His pulse is 120/min, respirations are 16/min, and blood pressure is
80/60 mm Hg. Compared with a healthy adult, which of the following findings is most likely in this patient?

(A)
(B)
(C)
(D)
(E)
(F)

Arterial Baroreceptor
Firing Rate

Systemic Vascular
Resistance

18

Pulmonary Vascular
Resistance

Systemic Capillary
Fluid Transfer
filtration
absorption
filtration
absorption
filtration
absorption

USMLE STEP1 SAMPLE TEST QUESTIONS


BLOCK 2, ITEMS 47-92
47.

A 37-year-old woman has blurred, double vision


8 hours after eating home-preserved peppers. Six
hours later, she has dysphagia, dry mouth and
eyes, progressive weakness of the arms and legs,
and urinary retention. She is awake and alert.
Which of the following is the most likely
mechanism of these adverse effects?
(A)
(B)
(C)
(D)
(E)

48.

Antagonism of muscarinic receptors


Antagonism of nicotinic receptors
Inhibition of acetylcholine release
Inhibition of cholinesterase activity
Inhibition of G proteins

51.

Abolished
Decreased but not abolished
Increased
Unchanged

Serum
HCO3

Arterial Blood
pH
PCO2

A 31-year-old woman comes to the physician


because of a 2-week history of malaise, nausea,
vomiting, and decreased appetite. She is a known
user of intravenous heroin. She appears
chronically ill. She is 165 cm (5 ft 5 in) tall and
weighs 47 kg (103 lb); BMI is 17 kg/m2. Her
temperature is 36.7C (98.1F), pulse is 90/min,
respirations are 18/min, and blood pressure is
114/68 mm Hg. Physical examination shows
scleral icterus and a liver span of 16 cm. The
spleen is not palpable. Serum studies show:
Total bilirubin
AST
ALT
HIV antibody
Hepatitis B surface antigen
Hepatitis B surface antibody
Anti-hepatitis B core antibody
Hepatitis B DNA
Anti-hepatitis C virus
Hepatitis C RNA

A 70-year-old man is brought to the emergency


department by his wife because of fever and
shortness of breath for 2 days. He underwent an
oral surgical procedure 6 weeks ago. His
respirations are 22/min, and blood pressure is
140/60 mm Hg. A soft diastolic murmur is heard.
The diagnosis of bacterial endocarditis is made.
Gentamicin therapy is initiated. This patient is at
increased risk for developing which of the
following as a result of this therapy?
(A)
(B)
(C)
(D)
(E)

A 26-year-old woman is brought to the


emergency department 3 hours after ingesting
approximately 50 tablets of aspirin in a suicide
attempt. She is nauseated, confused, and sleepy.
Her pulse is 130/min, respirations are 30/min,
and blood pressure is 100/60 mm Hg. Which of
the following sets of laboratory values is most
likely on evaluation of blood obtained before
treatment?

(A)
(B)
(C)
(D)
(E)

Vascular control is studied in an intact hind


extremity of an anesthetized experimental
animal. After a normal control period, the blood
flow to the extremity is completely occluded for
1 minute. When the occlusion is released, blood
flow increases abruptly and exceeds the control
value for several minutes (reactive hyperemia).
After an appropriate recovery period, the
procedure is repeated and the extremity is
actively exercised during the occlusion period.
Which of the following best describes the
reactive hyperemia after the second occlusion
compared with that after the first occlusion?
(A)
(B)
(C)
(D)

49.

50.

3.2 mg/dL
774 U/L
820 U/L
negative
negative
positive
positive
negative
positive
positive

Which of the following is the most likely


outcome of this patient's infection?
(A) Complete resolution of infection
(B) Latent infection with intermittent
viremia
(C) Lifelong persistent infection
(D) Patient death from acute infection

Cardiac ischemia
Hearing loss
Hyperglycemia
Lung infection
Torsades de pointes

19

52.

A 12-year-old boy is brought to the physician by


his mother because of a 1-month history of pain
below the left knee. His mother says, "He can
usually walk around, but he hasn't been able to
play in any of his soccer games since this all
began." Examination of the left knee shows
warmth, swelling, and tenderness. An x-ray of
the knee is shown. Which of the following
structures is attached to the abnormal anterior
tibial area?
(A)
(B)
(C)
(D)
(E)
(F)

53.

54.

(A) Arteriolar constriction and arteriolar


hypertension
(B) Arteriolar dilation and venous
hypertension
(C) Venous constriction and arteriolar
constriction
(D) Venous hypertension and incompetent
valves
(E) Venous hypertension and venous
constriction

Anterior cruciate ligament


Gastrocnemius muscle
Patellar ligament
Popliteus muscle
Posterior cruciate ligament
Soleus muscle

A 31-year-old woman with type 2 diabetes


mellitus comes to the physician because of an
oozing, foul-smelling wound on her foot for
2 days. Physical examination shows a 4-cm,
necrotizing wound with a purplish black
discoloration over the heel. Crepitant bullae
producing profuse amounts of serous drainage
are seen. A Gram stain of a tissue biopsy
specimen shows gram-positive rods. The causal
organism most likely produces which of the
following virulence factors?
(A)
(B)
(C)
(D)
(E)

A 64-year-old man comes to the physician


because of swelling in his feet for the past
2 years. He says that his skin is dry and itchy and
his feet "feel heavy." One of his legs is shown.
Which of the following is the most likely cause
of his condition?

Endotoxin
Fimbriae
Pneumolysin
Polysaccharide capsule
-Toxin
20

55.

A 55-year-old man is brought to the emergency department because of shortness of breath and confusion for
4 hours. He has hypertension and chronic kidney disease requiring hemodialysis. An ECG shows low voltage with
electrical alternans. Physical examination is most likely to show which of the following findings?

(A)
(B)
(C)
(D)
(E)
(F)

56.

Blood Pressure (mm Hg)


85/60
85/60
85/60
120/80
120/80
120/80

Pulse (/min)
120
120
120
80
80
80

A patient being treated with clindamycin for


aspiration pneumonia develops diarrhea. The
stool contains a toxin that kills cultured epithelial
cells. Stool culture grows an anaerobic grampositive rod. The same organism is cultured from
his bedpan. Which of the following is most
likely to sterilize the bedpan?

Jugular Venous Pressure


increased
increased
normal
increased
normal
normal

58.

57.

13.1 g/dL
39.7%
8500/mm3
65%
30%
5%
82.2 m3
20,000/mm3

A peripheral blood smear shows normal red cell


morphology; a bone marrow smear shows
mature megakaryocytic hyperplasia. Which of
the following is the most likely diagnosis?

A 15-year-old girl who is a ballet dancer has not


had a menstrual period for the past 3 months.
Menses were previously regular at 29-day
intervals. She has lost weight over the past year;
her weight is 70% of that expected for her
height. She is afebrile and has purpuric lesions
on her extremities and trunk. Platelet, absolute
neutrophil, and lymphocyte counts are below the
reference range. She has macrocytic anemia. The
most likely cause of these symptoms is a
deficiency of which of the following nutrients?
(A)
(B)
(C)
(D)
(E)
(F)
(G)
(H)
(I)
(J)
(K)
(L)
(M)

An otherwise healthy 26-year-old woman has


had petechiae on her legs during the last 24
hours. Laboratory studies show:
Hemoglobin
Hematocrit
Leukocyte count
Neutrophils
Lymphocytes
Monocytes
Mean corpuscular volume
Platelet count

(A) Boiling for 45 minutes


(B) Exposure to benzalkonium chloride for
1 hour
(C) Exposure to ethyl alcohol for 1 hour
(D) Exposure to saturated steam (121C) for
15 minutes
(E) Heating in an oven at 150C for 30
minutes

Pulsus Paradoxus
increased
normal
normal
increased
increased
normal

(A)
(B)
(C)
(D)
(E)
(F)
(G)

59.

Folic acid
Iron
Linoleic acid
Magnesium
Niacin
Protein
Vitamin A
Vitamin B6 (pyridoxine)
Vitamin C
Vitamin D
Vitamin E
Vitamin K
Zinc

A 64-year-old man with non-Hodgkin lymphoma


comes to the physician because of a 3-week
history of progressive numbness in his hands and
feet and weakness in his legs when he stands. He
received his third course of chemotherapy
4 weeks ago. Physical examination shows
areflexia. Which of the following drugs is the
most likely cause of these adverse effects?
(A)
(B)
(C)
(D)
(E)
(F)
(G)

21

Acute megakaryocytic leukemia


Acute myelogenous leukemia
Aplastic anemia
Immune thrombocytopenic purpura
Epstein-Barr viral infection
Papovavirus infection
Thrombotic thrombocytopenic purpura

Bleomycin
Cyclophosphamide
Cytarabine
Doxorubicin
Fluorouracil
Methotrexate
Vincristine

60.

A 36-year-old man with profound intellectual disability is brought to the physician by staff at his facility because of
increasing abdominal girth during the past 2 weeks. He is unable to speak, and no medical history is currently
available. Physical examination shows a protuberant abdomen with a fluid wave and shifting dullness. There are no
signs of trauma to the area. Laboratory studies show no abnormalities. A CT scan of the abdomen is shown. Fluid is
present in which of the following areas as indicated by the arrow?
(A)
(B)
(C)
(D)
(E)

61.

Epiploic foramen
Gastrosplenic ligament
Hepatorenal pouch (of Morison)
Omental bursa (lesser sac)
Sulcus pericolicus

A 66-year-old man is brought to the emergency


department by neighbors 1 hour after the sudden
onset of progressive confusion and sleepiness.
He had just seen the physician 1 week before,
and he appeared cheerful and his usual sharp self
until he was found wandering in his neighbor's
yard the day of admission. He appears sleepy,
but he is arousable. He is oriented to person, but
not to place or time. He recalls two of three
objects after 5 minutes. He says, "There are so
many little people here. What sort of place is
this?" He then falls back asleep. Which of the
following is the most likely cause of this patient's
condition?
(A)
(B)
(C)
(D)
(E)

62.

A 30-year-old man is brought to the emergency


department 30 minutes after being stung by
several wasps. He is confused and has difficulty
breathing. His temperature is 38C (100.4F),
pulse is 122/min, respirations are 34/min, and
blood pressure is 80/40 mm Hg. Physical
examination shows dry skin and decreased
capillary refill. There are multiple erythematous,
inflamed marks on the back and 1+ pitting
edema of the ankles. In addition to the
administration of 0.9% saline, the most
appropriate next step in management is
administration of which of the following?
(A)
(B)
(C)
(D)
(E)
(F)

Acute paranoid schizophrenia


Arsenic poisoning
Dementia, Alzheimer type
New medication regimen
Tertiary syphilis

22

Atropine
Captopril
Epinephrine
Losartan
Methacholine
Whole blood

65.

A previously healthy 19-year-old woman comes


to the physician because of a 3-day history of
fever, fatigue, and sore throat. She lives with a
roommate who has a cat. Her temperature is
37.8C (100F). Physical examination shows
mildly tender cervical and submental adenopathy
and pharyngitis. There is mild splenomegaly.
Laboratory studies show:
Hemoglobin
Hematocrit
Leukocyte count
Platelet count
Heterophile antibody titer

13.3 g/dL
43%
12,500/mm3
250,000/mm3
positive

The most likely cause of this patient's condition


is infection with which of the following?

63.

A 42-year-old woman is brought to the


emergency department because of double vision
that began 20 minutes after she fell from her
horse and landed on the left side of her face.
Examination of the face shows ecchymoses over
the left zygomatic arch. A CT scan of the head is
shown. Which of the following arteries is at
greatest risk for injury in this patient?
(A)
(B)
(C)
(D)
(E)

64.

(A)
(B)
(C)
(D)
(E)

66.

Facial
Frontal
Infraorbital
Lacrimal
Ophthalmic

(A)
(B)
(C)
(D)
(E)

A previously healthy 3-month-old boy is brought


to the physician because of a runny nose and a
dry cough for 2 days. Physical examination
shows tachypnea, a nasal discharge, and
wheezing. An x-ray of the chest shows
hyperexpansion but no infiltrates. The causal
virus was most likely transmitted by which of the
following routes?
(A)
(B)
(C)
(D)
(E)

Over 1 year, a study is conducted to assess the


antileukemic activity of a new tyrosine kinase
inhibitor in patients with chronic myeloid
leukemia in blast crisis. All patients enrolled in
the study are informed that they would be treated
with the tyrosine kinase inhibitor. They are
assigned to successive dose cohorts of 300 to
1000 mg/day of the drug. Six to eight patients
are assigned to each dose. Treatment efficacy is
determined based on the results of complete
blood counts and bone marrow assessments
conducted regularly throughout the study. This
study is best described as which of the
following?

67.

Case-control study
Crossover study
Open-labeled clinical trial
Randomized clinical trial
Single-blind, randomized, controlled
trial

Blood transfusion
Ingestion of contaminated formula
Inoculation onto mucous membranes
Insect bite
Transplacental transfer

Six healthy subjects participate in a study of


muscle metabolism during which hyperglycemia
and hyperinsulinemia is induced. Muscle biopsy
specimens obtained from the subjects during the
resting state show significantly increased
concentrations of malonyl-CoA. The increased
malonyl-CoA concentration most likely directly
inhibits which of the following processes in
these subjects?
(A)
(B)
(C)
(D)
(E)
(F)

23

Bartonella henselae
Cytomegalovirus
Epstein-Barr virus
Rhinovirus
Toxoplasma gondii

Fatty acid oxidation


Fatty acid synthesis
Gluconeogenesis
Glycogenolysis
Glycolysis
Oxidative phosphorylation

68.

A 6-year-old boy is brought to the physician by


his parents because of a 3-day history of fever,
headache, and cough productive of a green, foulsmelling discharge that also exits from his nose.
He has had repeated episodes of similar
symptoms during the past 4 years. He appears
pale and lethargic. His height and weight are
both below the 10th percentile. Coarse rhonchi
are heard bilaterally. An x-ray of the chest shows
scattered peripheral opacities, dilated and
thickened airways consistent with bronchiectasis,
and a cardiac apex that is directed toward the
right. The most likely cause of his recurrent
infections is a dysfunction of which of the
following cell types?
(A)
(B)
(C)
(D)
(E)
(F)
(G)
(H)
(I)
(J)

69.

71.

Hematocrit
Hemoglobin
Mean corpuscular volume

(A)
(B)
(C)
(D)
(E)

Alveolar capillary endothelial cell


Alveolar macrophage
Chondrocyte
Ciliated columnar epithelial cell
Clara cell
Goblet cell
Kulchitsky cell
Squamous epithelial cell
Type I pneumocyte
Type II pneumocyte

72.

A 45-year-old man has abnormal circadian


variation in body temperature, disruption of the
sleep-wake cycle, and an impaired nocturnal
surge of secretion of melatonin. An MRI of the
brain is most likely to show a lesion involving
which of the following nuclei?
(A)
(B)
(C)
(D)
(E)

Accessory optic
Lateral preoptic
Pretectal
Suprachiasmatic
Supraoptic

Antiplatelet antibody syndrome


Antithrombin III deficiency
Factor V Leiden mutation
Protein C deficiency
Protein S deficiency

A 12-year-old girl is admitted to the hospital


because of marked shortness of breath, an
erythematous rash, and painful, swollen hip and
knee joints. She is agitated. A chest x-ray shows
an enlarged heart and changes consistent with
pulmonary edema. Intractable congestive heart
failure develops, and she dies on the second
hospital day. This child most likely had a recent
history of which of the following?
(A)
(B)
(C)
(D)
(E)

24

Acquired hemolytic anemia


Chronic blood loss
Folic acid deficiency
-Thalassemia minor
Pernicious anemia

A 42-year-old woman is brought to the


emergency department 4 hours after the onset of
severe shortness of breath. She has no recent
history of trauma, hospital admission, or
operations. She had an episode of deep venous
thrombosis 10 years ago that required treatment
in the hospital. Her respirations are 34/min.
Pulse oximetry on room air shows an oxygen
saturation of 65%. A helical CT scan shows a
large filling defect in the right pulmonary artery.
Which of the following hypercoagulability
disorders is the most likely underlying cause of
these findings?
(A)
(B)
(C)
(D)
(E)

Adrenal adenoma
Choriocarcinoma
Ectopic pregnancy
Pituitary insufficiency
A second noninvasive mole
73.

70.

28%
9 g/dL
70 m3

Which of the following is the most likely basis


for these findings?

A 23-year-old woman has a progressive increase


in her serum -human chorionic gonadotropin
(-hCG) concentrations during an 8-week period.
A hydatidiform mole is removed, but the -hCG
concentration continues to increase. Which of the
following is the most likely diagnosis?
(A)
(B)
(C)
(D)
(E)

A 75-year-old woman has increasing shortness


of breath on exertion. Findings on physical
examination are unremarkable. X-rays of the
chest show no abnormalities of the heart or
lungs. Pertinent laboratory findings include:

Cyanosis with chest pain


Jaundice
Meningitis
Pharyngitis
Skin infection

74.

A population-based, case-control study is conducted to assess whether there is an association between statin use and
incidence of colorectal cancer. A total of 900 participants are enrolled: 400 patients who were diagnosed with
colorectal cancer between 1998 and 2004, and 500 healthy participants matched for age, sex, and ethnicity. A
structured interview is used to determine statin use in the two groups. Results are shown:

Statin Use

Positive
Negative
Total

Colon Cancer
Present
Absent
100
200
300
300
400
500

Total
300
600
900

Which of the following is the estimated odds ratio of colon cancer in statin-treated patients compared with patients
with no statin treatment?
(A)
(B)
(C)
(D)
(E)
(F)
(G)
(H)

75.

1.0
0.5
0
0.5
0.67
0.75
1.3
2

A 5-year-old child with short stature is being


evaluated for delayed dentition and excessive
caries. Examination of the chest shows pectus
carinatum and bead-like enlargement of the
costochondral junctions. Which of the following
findings is most likely on histologic examination
of a section of bone?

77.

(A) Absence of cartilage in the epiphyseal


plates
(B) Absence of osteoblasts
(C) Enlarged osteoclasts with an increased
number of nuclei
(D) Increased proportions of osteoid

76.

(A) Decreased entry of glucose into the


muscle cells
(B) Decreased production of glucose from
the liver
(C) Decreased secretion of insulin from the
pancreas
(D) Decreased speed of carbohydrate
absorption from the intestines
(E) Increased entry of glucose into the
muscle cells
(F) Increased production of glucose from
the liver
(G) Increased secretion of insulin from the
pancreas
(H) Increased speed of carbohydrate
absorption from the intestines

A 12-year-old boy is brought to the physician by


his father because of redness and swelling of his
left foot for 24 hours. Three days ago, the boy
scraped his foot while wading in a drainage
ditch. Examination of the left foot shows a
purulent abrasion with edema, erythema, and
tenderness on the lateral side. Infection is most
likely to next spread from the lateral side of the
foot to the regional lymph nodes in which of the
following areas?
(A)
(B)
(C)
(D)
(E)

A 52-year-old man with recently diagnosed type


2 diabetes mellitus comes to the physician for a
follow-up examination. Physical examination
shows no abnormalities. Laboratory studies show
an increased hemoglobin A1c despite patient
compliance
with
diet
and
exercise
recommendations. Treatment with a sulfonylurea
is started. Which of the following is most likely
to occur in this patient?

Lateral surface of the thigh


Medial malleolus, posteriorly
Popliteal fossa
Sole of the foot
Superficial inguinal area

25

78.

A 2-year-old boy is brought to the emergency department because of shortness of breath and left-sided abdominal
pain for 3 hours. He appears pale. Physical examination shows hypotension and tachycardia. There is splenomegaly
with the spleen tip palpated 8 cm below the left costal margin. Laboratory studies show:
Hemoglobin
Hematocrit
Leukocyte count
Platelet count

5.1 g/dL (N=12.114.9)


16% (N=37%44.4%)
4500/mm3 (N=400011,500)
87,000/mm3 (N=150,000400,000)

A photomicrograph of a Wright-stained peripheral blood smear is shown. Which of the following is the most likely
cause of this patient's current condition?
(A)
(B)
(C)
(D)
(E)

79.

Aplastic crisis
Autoimmune hemolysis
Congestive heart failure
Salmonellal sepsis
Splenic sequestration

A 73-year-old woman comes to the physician


because of a 2-month history of diffuse weakness
and tingling of her arms and legs. Neurologic
examination shows weakness of the extensor and
flexor muscles of the lower extremities. Knee
and ankle deep tendon reflexes are exaggerated.
Sensation to vibration and position is decreased
in all extremities, but the decrease is more
prominent in the lower extremities than in the
upper extremities. This patient most likely has a
deficiency of which of the following vitamins?
(A)
(B)
(C)
(D)
(E)

80.

In a cohort study of elderly women, the relative


risk ratio for hip fractures among those who
exercise regularly is 1.2 (95% confidence
interval of 1.1 to 1.8). Which of the following is
the most appropriate conclusion about the effect
of regular exercise on the risk for hip fracture?
(A) Statistically nonsignificant increase in
risk
(B) Statistically nonsignificant overall
decrease in risk
(C) Statistically significant overall decrease
in risk
(D) Statistically significant overall increase
in risk

Niacin
Vitamin B1 (thiamine)
Vitamin B2 (riboflavin)
Vitamin B6 (pyridoxine)
Vitamin B12 (cyanocobalamin)

26

81.

A 32-year-old man with non-Hodgkin lymphoma


comes to the physician 6 days after finishing the
initial chemotherapy regimen. His leukocyte
count is 1600/mm3, indicating greater bone
marrow suppression than expected. When
questioned, the patient says that he has been
taking Madagascar periwinkle as an herbal
remedy for his condition. He obtains this
substance from an herbalist. Which of the
following is the most appropriate response by the
physician?

84.

(A)
(B)
(C)
(D)
(E)
(F)

(A) Ask the patient to stop using the herbal


supplement because supplements are
generally ineffective
(B) Continue the patient's chemotherapy
(C) Explain the adverse effects this herbal
supplement has on the patient's
treatment
(D) Report the herbalist to the Food and
Drug Administration
(E) Suggest that the patient take daily
multivitamin and protein
supplements in addition to the
herbal supplement

82.

83.

85.

Ethinyl estradiol
Leuprolide
Medroxyprogesterone
Nandrolone
Tamoxifen

86.

A 55-year-old man comes to the physician


because of a 2-week history of recurrent,
widespread
blister
formation.
Physical
examination shows lesions that are most
numerous in the flexural areas including the
axillae and groin. The blisters do not break
easily, and there are no oral lesions. These
blisters are most likely the result of adhesion
failure involving which of the following?
(A)
(B)
(C)
(D)
(E)

27

Catfish
Cod
Salmon
Swordfish
Tilapia

A 16-year-old boy is admitted to the emergency


department because of a knife wound to the left
side of his chest. An x-ray of the chest shows an
air-fluid level in the left side of the chest, partial
collapse of the left lung, and elevation of the
stomach bubble. The mediastinum is in the
midline. Which of the following is the most
likely diagnosis?
(A)
(B)
(C)
(D)
(E)
(F)

Basement membrane
Dermal papillae
Langerhans cells
Melanocytes
Merkel cells

Case-control
Case series
Crossover
Cross-sectional
Historical cohort
Randomized clinical trial

A 33-year-old woman comes to the physician


2 days after the result of a home pregnancy test
was positive. The result of a serum pregnancy
test in the office is positive. She tells the
physician that she enjoys seafood, but she has
heard that some types "can be bad for the baby."
It is most appropriate for the physician to
recommend that the patient avoid which of the
following species of fish during her pregnancy?
(A)
(B)
(C)
(D)
(E)

A 16-year-old girl has hirsutism, deepening of


the voice, and cessation of menses. She swims in
competitions. Which of the following drugs is
most likely to have caused these findings?
(A)
(B)
(C)
(D)
(E)

A study is designed to evaluate the feasibility of


acupuncture in children with chronic headaches.
Sixty children with chronic headaches are
recruited for the study. In addition to their usual
therapy, all children are treated with acupuncture
three times a week for 2 months. Which of the
following best describes this study design?

Hemopneumothorax, not under tension


Hemothorax, not under tension
Pneumothorax, not under tension
Tension hemopneumothorax
Tension hemothorax
Tension pneumothorax

87.

A 45-year-old woman has a 6-month history of progressive shortness of breath on exertion. She does not smoke.
Pulmonary function findings are shown (values are given as % of predicted normal):
Vital capacity
Forced expiratory volume in 1 second (FEV1)
Diffusing capacity for carbon monoxide
Maximum voluntary ventilation

60
70
50
60

Which of the following most likely explains her limited ability to increase ventilation?
(A)
(B)
(C)
(D)
(E)

88.

A 74-year-old man with urinary frequency and


urgency has benign prostatic hyperplasia. He
refuses operative intervention but agrees to a trial
of finasteride therapy. During the trial, synthesis
of which of the following substances is most
likely to be inhibited?
(A)
(B)
(C)
(D)
(E)

89.

Airway obstruction
Decreased activation of pulmonary juxtacapillary (J) receptors
Decreased lung compliance
Depression of central chemoreceptors
Depression of peripheral chemoreceptors

90.

Androstenedione
Dihydrotestosterone
Estradiol
Estrone
Testosterone

(A)
(B)
(C)
(D)
(E)

A 30-year-old man with peptic ulcer disease


suddenly develops pain, redness, and swelling of
his right first metatarsophalangeal joint. There is
no history of injury. Serum uric acid
concentration is 8 mg/dL. Examination of joint
aspirate shows birefringent crystals. Which of
the following drugs is most appropriate to treat
the acute symptoms in this patient?
(A)
(B)
(C)
(D)
(E)

A 55-year-old man who is a business executive


is admitted to the hospital for evaluation of
abdominal pain. He is polite to the physician but
berates the nurses and other staff. The patient's
wife and two of his three adult children arrive for
a visit. The patient says with disgust that the
missing child is and always has been worthless.
Which of the following is the most likely
explanation for this patient's behavior?

Allopurinol
Colchicine
Morphine
Probenecid
Sulfinpyrazone

28

Countertransference
Projection
Projective identification
Reaction formation
Splitting

91.

A 14-year-old girl is brought to the physician by her mother because of a 2-month history of heavy vaginal bleeding
during menstrual periods. She has had episodes of excessive periodontal bleeding while brushing her teeth and easy
bruising for 6 years. She also had an episode of extended bleeding after a tooth extraction 4 years ago. Her mother
and brother have had similar symptoms. Physical examination shows patchy ecchymoses over the upper and lower
extremities. Laboratory studies show:
234,000/mm3
17 min
12 sec (INR=1)
46 sec

Platelet count
Bleeding time
Prothrombin time
Partial thromboplastin time
Which of the following is the most likely diagnosis?
(A)
(B)
(C)
(D)
(E)
(F)

92.

Factor VII (proconvertin) deficiency


Factor X (Stuart factor) deficiency
Factor XII (Hageman factor) deficiency
Hemophilia A
Vitamin K deficiency
von Willebrand disease

A previously healthy 40-year-old man is brought to the emergency department because of constant substernal chest
pain for 12 hours that is exacerbated by coughing and inspiration. The pain is relieved with sitting up and leaning
forward. There is no family history of heart disease. His temperature is 38C (100.4F), pulse is 120/min, and blood
pressure is 110/60 mm Hg. The lungs are clear to auscultation. Cardiac examination shows distant heart sounds. An
ECG shows diffuse ST-segment elevation in all leads. An x-ray of the chest shows normal findings. The most likely
cause of his condition is injury to which of the following tissues?
(A)
(B)
(C)
(D)
(E)

Aortic intima
Esophageal sphincter
Myocardium
Pericardium
Pleura

29

USMLE STEP1 SAMPLE TEST QUESTIONS


BLOCK 3, ITEMS 93-138
93.

A 17-year-old boy comes to the emergency


department because of severe thirst and
weakness and a 4-kg (8.8-lb) weight loss
over the past 36 hours. He began having
voluminous painless watery diarrhea on the
airplane while returning from a trip to
Thailand 36 hours ago. He has not vomited.
While supine, pulse is 110/min and blood
pressure is 110/60 mm Hg. While standing,
pulse is 170/min and blood pressure is
70/40 mm Hg. His abdomen is nontender
and bowel sounds are increased. Which of
the following treatments is most appropriate
at this time?
(A)
(B)
(C)
(D)
(E)
(F)

94.

Ciprofloxacin
Doxycycline
Exploratory laparotomy
Potassium chloride
Rehydration
Trimethoprim-sulfamethoxazole

Amitriptyline
Diazepam
Fluoxetine
Haloperidol
Levodopa

97.

A 66-year-old man has become increasingly


short-tempered with his wife. He has
diarrhea, weight loss, and weakness in the
proximal muscles. He has atrial fibrillation
and tachycardia. Which of the following is
the most likely diagnosis?
(A)
(B)
(C)
(D)
(E)

After being severely beaten and sustaining a


gunshot wound to the abdomen, a 42-yearold woman undergoes resection of a
perforated small bowel. During the
operation, plastic reconstruction of facial
fractures, and open reduction and internal
fixation of the left femur are also done.
Thirty-six hours postoperatively, she is
awake but not completely alert. She is
receiving intravenous morphine via a
patient-controlled pump. She says that she
needs the morphine to treat her pain, but she
is worried that she is becoming addicted.
She has no history of substance use disorder.
She drinks one to two glasses of wine
weekly. Which of the following initial
actions by the physician is most appropriate?
(A) Reassure the patient that her chance
of becoming addicted to
narcotics is minuscule
(B) Maintain the morphine, but
periodically administer
intravenous naloxone
(C) Switch the patient to oral
acetaminophen as soon as she
can take medication orally
(D) Switch the patient to intramuscular
lorazepam
(E) Switch the patient to intravenous
phenobarbital

A 22-year-old man comes to the emergency


department because of the recent onset of
torticollis
and
uncontrollable
facial
grimacing. He began therapy with a new
drug 24 hours ago. Which of the following
drugs is the most likely cause?
(A)
(B)
(C)
(D)
(E)

95.

96.

A 49-year-old woman is found to have mild


proteinuria and pyuria on routine screening.
Urine specific gravity is 1.000. Culture of
the urine grows no organisms. She has taken
large doses of combination over-the-counter
analgesic preparations for 10 years due to a
low back injury. Which of the following
renal abnormalities is most likely in this
patient?
(A)
(B)
(C)
(D)
(E)

Congestive heart failure


Cushing syndrome
Hyperthyroidism
Mitral valve prolapse
Pheochromocytoma

30

Acquired cystic disease


Acute glomerulonephritis
Hyperplastic arteriolitis
Nephrolithiasis
Papillary necrosis

98.

A previously healthy 26-year-old patient develops shortness of breath over several hours during a mountain
climb at 5000 m (16,404 ft). There is no history of trauma. On examination, the point of cardiac apical
impulse is not displaced and there are widespread crackles throughout both lung fields. Which of the
following x-rays of the chest shown best represents this patient?

31

101.

A 35-year-old man comes to the physician


because of pain and swelling of his right arm
where he scraped it on a tree branch 2 days
ago. His temperature is 38.3C (101F).
Examination of the right forearm shows
edema around a fluctuant erythematous
lesion at the site of trauma. The area is
extremely tender to palpation. Which of the
following is most likely the primary
mechanism of the development of edema in
this patient?
(A) Degranulation of eosinophils
(B) Disruption of vascular basement
membranes
(C) Increased hydrostatic pressure
(D) Release of thromboxane
(E) Separation of endothelial junctions

102.
99.

During an experiment, an investigator finds


a point mutation (CGGTGG) in the gene
encoding the protein kinase regulatory
subunit. The genetic code is shown in the
figure. This mutation is most likely to alter
the amino acid sequence with which of the
following changes?
(A)
(B)
(C)
(D)
(E)

100.

ArgTrp
GlyArg
GlyTrp
ThrGly
TrpArg

(A) Cardiac tamponade


(B) Embolus to the right middle
cerebral artery
(C) Necrosis of the myocardium
(D) Rupture of the papillary muscle
(E) Ventricular fibrillation

A 6-day-old breast-fed boy is brought to the


emergency department by his mother
because of poor weight gain and irritability
since delivery, and a 2-hour history of
vomiting. Physical examination shows
jaundice and hepatomegaly. A reducing
substance test result of the urine is positive,
and a glucose oxidase test result is negative.
The concentration of which of the following
metabolites in liver is most likely increased
in this patient?
(A)
(B)
(C)
(D)

A 72-year-old man collapses while playing


golf. He has a 5-year history of angina and
type 2 diabetes mellitus. Paramedics arrive
in 10 minutes. Examination shows no
respirations or blood pressure; an ECG
shows
asystole.
Cardiopulmonary
resuscitation is attempted for 10 minutes
without success. Which of the following is
the most likely cause of death in this
patient?

103.

A 60-year-old woman with reflux


esophagitis comes to the physician for a
follow-up examination. She has been
undergoing proton-pump inhibitor therapy
for the past month and shows clinical
improvement. Upper endoscopy is done. A
biopsy specimen of the stomach is most
likely to show which of the following?
(A)
(B)
(C)
(D)
(E)

Fructose 1,6-bisphosphate
Galactose 1-phosphate
Glucose 1-phosphate
Glucose 6-phosphate

32

Acute inflammation
Gastric atrophy
Intramucosal smooth muscle
Lymphoid nodules
Parietal cell hyperplasia

104.

A 40-year-old woman receives an intravenous infusion of drug X that selectively constricts the efferent
arterioles in her kidneys. Following the infusion, total cardiac output and renal afferent arteriolar tone are
unchanged, but renal efferent arteriolar tone and total renal vascular resistance have both increased. Which
of the following sets of changes most likely occurred following the infusion of drug X?
Glomerular Filtration Rate

(A)
(B)
(C)
(D)
(E)

105.

Filtration Fraction

A 28-year-old woman comes to the


physician because of a 6-month history of
intermittent feelings of progressive
overwhelming fear and apprehension,
restlessness, and palpitations. She says
that she often feels tense, and she has
difficulty concentrating because she is
unable to stop thinking about the things
that worry her. Physical examination
shows no abnormalities. A selective
serotonin reuptake inhibitor (SSRI) is
prescribed for long-term management.
Which of the following is most
appropriate to provide immediate relief
for this patient until the effects of the
SSRI treatment occur?
(A)
(B)
(C)
(D)
(E)
(F)

107.

11.3 g/dL
34%
5.2 million/mm3
65 m3

Follow-up laboratory studies show that


the serum iron concentration and ironbinding capacity are within the reference
ranges.
Hemoglobin
electrophoresis
shows increased hemoglobin A2 (5%).
Which of the following is the most likely
diagnosis?

Alprazolam
Amitriptyline
Diphenhydramine
Haloperidol
Phenobarbital
Tramadol

(A)
(B)
(C)
(D)
(E)

A healthy 28-year-old woman comes to


the physician for advice on losing weight.
She is 150 cm (4 ft 11 in) tall and weighs
56 kg (124 lb); BMI is 25 kg/m2. Physical
examination
shows
no
other
abnormalities. The physician recommends
a diet that will restrict her daily intake by
500 kilocalories. Which of the following
processes is most likely to increase in this
patient as a result of following this diet?
(A)
(B)
(C)
(D)
(E)

A 25-year-old woman has a routine, preemployment


physical
examination.
Laboratory studies include:
Hemoglobin
Hematocrit
Erythrocyte count
Mean corpuscular volume

108.
106.

Renal Blood Flow

Adipocyte glucose uptake


Cerebral ketone utilization
Hepatic lipid oxidation
Muscle glucose uptake
Resting energy expenditure

An 18-year-old woman comes to the


physician for a health maintenance
examination. She has not had major
medical illnesses. She takes no
medications. She does not smoke
cigarettes, drink alcohol, or use illicit
drugs. Physical examination shows diffuse
brownish yellow discoloration of all teeth.
Which of the following most likely
occurred during childhood to cause this
finding?
(A)
(B)
(C)
(D)
(E)
(F)

33

Anemia of chronic disease


Iron deficiency anemia
Sideroblastic anemia
-Thalassemia minor
-Thalassemia minor

Amelogenesis imperfecta
Dentinogenesis imperfecta
Rh incompatibility
Syphilis
Tetracycline use
Vitamin D deficiency

109.

A 16-year-old boy comes to the physician


because of a rash on his left inner thigh
that first appeared 2 days after he returned
from a hunting trip with friends in
Minnesota. A photograph of the rash is
shown. Without treatment, this patient is
at increased risk for which of the
following?
(A)
(B)
(C)
(D)
(E)

110.

111.

Carditis
Glomerulonephritis
Hepatitis
Pancreatitis
Thrombocytopenia

A 37-year-old woman comes to the


physician because of a 2-month history of
pain with movement of her hands and
feet. Physical examination shows warmth
and swelling of the metacarpophalangeal
and
metatarsophalangeal
joints.
Laboratory studies show increased titers
of antibodies to Fc component of IgG and
a negative antinuclear antibody test result.
A drug is prescribed that binds to tumor
necrosis factor- (TNF-) and blocks its
interaction
with cell-surface
TNF
receptors. Her symptoms improve within
1 month. She is most likely receiving
treatment with which of the following
drugs?
(A)
(B)
(C)
(D)
(E)

A 63-year-old homeless man is brought to


the emergency department 1 hour after
police found him unresponsive. His
respirations are 30/min. Crackles are
heard over the left upper and the entire
right lung fields. Despite appropriate
lifesaving measures, he dies. A
photomicrograph of a section of the right
lung obtained at autopsy is shown. Which
of the following mediators is the most
likely cause of the position of the cell
indicated by the arrow?
(A)
(B)
(C)
(D)
(E)

112.

Adalimumab
Anakinra
Gold
Methotrexate
Prednisone

A 48-year-old man with AIDS has


chorioretinitis
with
flame-shaped
hemorrhages and large white patches.
Three weeks after starting therapy, his
vision is improved, but he has
granulocytopenia. Which of the following
antiviral agents is most likely responsible
for these findings?
(A)
(B)
(C)
(D)
(E)

34

Bradykinin
C5a
Histamine
Nitrous oxide
Prostaglandins

Amantadine
Foscarnet
Ganciclovir
Lamivudine (3TC)
Ribavirin

113.

A 30-year-old woman, gravida 3, para 3,


comes to the physician because of a 2month history of loss of urine when she
sneezes or laughs. Her three children were
all delivered vaginally at term. Physical
examination shows no abnormalities.
Which of the following nerve roots
supplies the muscle group that is
weakened in this patient?
(A)
(B)
(C)
(D)
(E)

114.

C1-2
L1-2
L4-5
S1-2
S3-4

A 36-year-old woman has been trying to


conceive for the past 2 years. Her menses
occur every 19 to 45 days. She has a past
history of some type of sexually
transmitted disease, but says she was
treated and cured. She is 163 cm (5 ft 4 in)
tall and weighs 109 kg (240 lb); BMI is
41 kg/m2. Physical examination including
a pelvic examination is unremarkable. An
endometrial biopsy is performed based on
the menstrual history and a negative
pregnancy test. The biopsy shows stromal
breakdown associated with proliferative
glands. Which of the following is the most
likely cause of her infertility?
(A)
(B)
(C)
(D)
(E)

115.

116.

(A) Refer the patient for behavior


therapy
(B) Restrict the patient's fluid intake
(C) Add lithium carbonate to the
medication regimen
(D) Administer furosemide therapy
(E) Begin mineralocorticoid therapy

117.

Anovulation
Chronic endometritis
Endometrial polyps
Endometriosis
Leiomyomata

A 1-day-old newborn is evaluated for


possible sepsis. Blood cultures grow
gram-positive cocci in pairs and chains
that agglutinate with group B antiserum.
The most likely epidemiologic risk factor
for this infection involves bacterial
colonization of which of the following?
(A)
(B)
(C)
(D)
(E)

A 26-year-old woman is brought to the


emergency department by her mother
1 hour after she had a generalized tonicclonic seizure at home. The mother states
that her daughter has been talking to
herself at all hours of the day and night
about being thirsty. The patient was found
to have schizophrenia 6 years ago. Her
only medication is haloperidol. She was
admitted to the hospital twice in the past
year for psychotic episodes. Physical
examination
shows
no
other
abnormalities. She is oriented to person
but not to place or time. Her serum
sodium concentration is 114 mEq/L on
arrival but returns to normal with
appropriate treatment. Which of the
following interventions is the most
appropriate next step?

A 29-year-old man is brought to the


physician for removal of a cast from his
left leg. He sustained a fracture of the left
lower extremity 6 weeks ago and was
immobilized in a cast that extended from
just below the knee to the foot. At the
time of injury, there was severe pain but
normal strength in the extremity. When
the cast is removed today, physical
examination shows a pronounced left
footdrop with paresthesia and sensory loss
over the dorsum of the left foot and lateral
leg. Injury to which of the following
nerves is the most likely cause of this
patient's condition?
(A)
(B)
(C)
(D)
(E)

Mother's vagina
Newborn's gastrointestinal tract
Newborn's nasopharynx
Placenta
Umbilical cord remnant

35

Common fibular (peroneal)


Femoral
Obturator
Sciatic
Tibial

118.

An investigator is studying the effect of the number of hours watching television (Factor A) on the
percent of hemoglobin A1c in people with type 2 diabetes mellitus. Two different variables, Factor A
and hemoglobin A1c, are compared. The results of the study indicate a correlation coefficient of +0.9.
Which of the following graphs shown best corresponds to these results?

119.

A 24-year-old woman comes to the


physician for a follow-up examination.
One week ago, she was treated in the
emergency
department
after
she
accidentally spilled hot grease on her left
leg while working at a fast-food
restaurant. Examination of the left lower
extremity shows a 7-cm, pink, soft,
granular,
edematous
wound.
The
formation of this tissue was most likely
caused by increased activity of which of
the following?

120.

(A) Complement C3b


(B) Glycosylation-dependent cell
adhesion molecule-1
(C) P-selectin
(D) Stromelysin
(E) Vascular endothelial growth
factor

A 10-year-old boy is brought to a new


physician by his parents for an initial
examination. The patient was born with
congenital glaucoma, hearing loss, and a
patent ductus arteriosus that has since
been surgically corrected. At the time of
birth, he also had purpura, jaundice, and
splenomegaly. He is in a special education
class for students with mental retardation.
He is at the 75th percentile for height and
weight, and 5th percentile for head
circumference. This patient's condition
was most likely caused by in utero
exposure to which of the following?
(A)
(B)
(C)
(D)
(E)

36

Cytomegalovirus infection
HIV infection
Rubella
Syphilis
Toxoplasmosis

123.

121.

122.

Specific gravity
Dipstick
Glucose
Blood
Nitrites
Microscopic examination
WBC
RBC

A 45-year-old woman is brought to the


physician because of a 4-month history of
muscle fatigue. She says that she has
difficulty initiating movements and
focusing her eyes. Physical examination
shows bilateral ptosis of the upper eyelids.
Repetitive nerve stimulation testing of a
motor nerve shows a 30% decrease in the
compound muscle action potential
amplitude. A CT scan of the chest is
shown;
the
arrow
indicates
an
abnormality. Which of the following is the
most likely cause of the muscle weakness
in this patient?
(A)
(B)
(C)
(D)
(E)

One day after a 10-km race, a previously


healthy 42-year-old man has dark urine.
Urinalysis shows:
1.010
negative
positive
negative
negative
negative

Which of the following is the most likely


cause of these findings?
(A)
(B)
(C)
(D)
(E)

124.

Mesothelioma of the pleura


Myasthenia gravis
Sarcoidosis
Small cell carcinoma of the lung
Squamous cell carcinoma of the
esophagus

An investigator is conducting a study of


hypertension in an experimental animal
model. Results show that hypertension is
induced when one renal artery is
constricted with a clamp, but both kidneys
remain intact. Which of the following best
explains the contribution of the normal
kidney to the hypertension in this
experimental model?
(A) Aldosterone-induced sodium and
water retention
(B) Angiotensin I-induced
vasoconstriction
(C) Angiotensin II-induced renal
vein constriction
(D) Expression of angiotensinconverting enzyme
(E) Increased renin secretion

A 68-year-old man comes to the physician


because of worsening nonproductive
cough and progressive shortness of breath
during the past 2 months. Three months
ago, he was able to walk an unlimited
distance without difficulty; now, he
becomes short of breath after walking one
block. He has had no fever, chest pain, or
leg swelling. He has atrial fibrillation and
hypertension treated with medications.
His temperature is 37.7C (99.8F), pulse
is 90/min, and respirations are 22/min.
Pulse oximetry on room air shows an
oxygen saturation of 92%. Diffuse
inspiratory crackles are heard bilaterally.
The remainder of the examination shows
no abnormalities. His plasma brain
natriuretic peptide concentration is within
the reference range. A chest x-ray shows
bilateral interstitial infiltrates. Which of
the following medications is the most
likely cause of these findings?
(A)
(B)
(C)
(D)
(E)

37

Acute glomerulonephritis
Hypovolemia
Renal infarct
Renal vein thrombosis
Rhabdomyolysis

Amiodarone
Digoxin
Lisinopril
Metoprolol
Procainamide

125.

A 2-week-old female newborn delivered at term is brought to the physician by her mother because of
an increasingly severe diaper rash since birth. No congenital anomalies were noted after delivery.
Physical examination shows a red and swollen umbilical remnant that has not separated. There are
ulcerations of the skin but no purulent exudate in the area of the diaper. A culture of one of the ulcers
grows Staphylococcus aureus. Despite antibiotic therapy, 1 month later she develops a perirectal
fissure, culture of which grows Escherichia coli but a smear of which shows scarce segmented
neutrophils. Laboratory studies now show:
Hemoglobin
Hematocrit
Mean corpuscular volume
Leukocyte count
Segmented neutrophils
Bands
Lymphocytes
Monocytes
Platelet count
Serum
IgA
IgG
IgM

12.7 g/dL
38%
98 m3
89,790/mm3
89%
6%
3%
2%
249,000/mm3
92 mg/dL
766 mg/dL
101 mg/dL

A peripheral blood smear shows normochromic, normocytic erythrocytes and leukocytes with normal
morphology. This patient most likely has which of the following conditions?
(A)
(B)
(C)
(D)
(E)

126.

Acute myelogenous leukemia


AIDS
Chdiak-Higashi syndrome
Common variable immunodeficiency
Leukocyte adhesion deficiency

A 52-year-old woman comes to the


physician because of a 1-month history of
headache, weakness, tingling of her
extremities, muscle cramping, and fatigue.
Her blood pressure is 170/110 mm Hg.
Physical examination shows no other
abnormalities. Laboratory studies show a
decreased serum potassium concentration,
metabolic alkalosis, and decreased plasma
renin
activity;
serum
sodium
concentration is within the reference
range.
Urine
catecholamine
concentrations are within the reference
range. Which of the following is the most
likely diagnosis?

127.

A 72-year-old man who is a retired


construction worker comes to the
physician because he has had a lesion on
his face for 3 months. Physical
examination shows a 6-mm, red, ulcerated
lesion with heaped borders. A biopsy
specimen of the lesion shows atypical,
dysplastic keratinocytes within the
epidermis and dermis. Which of the
following is the most likely diagnosis?
(A)
(B)
(C)
(D)
(E)

(A) Adrenal adenoma


(B) Focal segmental
glomerulosclerosis
(C) Hypothalamic tumor
(D) Juxtaglomerular cell tumor
(E) Renal artery stenosis

38

Actinic keratosis
Discoid lupus erythematosus
Melanoma
Mycosis fungoides
Squamous cell carcinoma

128.

An 84-year-old woman who resides in an


assisted living facility is brought to the
emergency department because of fever
and cough for 1 week. The cough has
been productive of foul-smelling, yellowgreen sputum for 24 hours. She has a 2year history of dementia, Alzheimer type.
Her temperature is 38.5C (101.3F),
pulse is 80/min, respirations are 20/min,
and blood pressure is 116/66 mm Hg.
Coarse inspiratory crackles are heard over
the right lung field. Laboratory studies
show a leukocyte count of 13,500/mm3
(72% segmented neutrophils, 8% bands,
1% eosinophils, 16% lymphocytes, and
3% monocytes). A CT scan shows a
cavitary lesion in the superior segment of
the right lower lobe. The lesion has a thick
wall and an irregular peripheral margin;
there is no displacement of the adjacent
bronchovascular bundle. Which of the
following is the most likely cause of the
lung lesion in this patient?

130.

(A)
(B)
(C)
(D)
(E)

131.

(A) Antecedent viral pneumonia


(B) Aspiration of gastric contents
(C) Bronchial obstruction by
metastatic carcinoma
(D) Lung infarction secondary to
arterial thrombosis
(E) Primary carcinoma of the lung
(F) Secondary infection of a
congenital lung cyst
(G) Septic embolism from an
extrapulmonary site

129.

Atrial septal defect


Bicuspid aortic valve
Coarctation of the aorta
Patent ductus arteriosus
Tetralogy of Fallot

A study is conducted to assess the


relationship between serum protein YY
(PYY) concentrations and obesity. A total
of 24 subjects with BMIs ranging from 17
to 40 kg/m2 are enrolled in the study. A
fasting serum PYY concentration is
determined for each patient. The
investigators report that the correlation
coefficient between the two parameters is
0.84 (p<0.001). Based on these results,
which of the following is the most
accurate conclusion?
(A) A decreased serum PYY
concentration is a cause of
obesity
(B) The likelihood of a correlation
between high BMIs and
serum PYY concentrations is
0.16
(C) Obesity is not related to serum
PYY concentrations
(D) The slope of the line showing the
relationship between serum
PYY concentrations and
BMIs is 0.84
(E) Subjects with lower BMIs have
higher serum PYY
concentrations

A 20-year-old man comes to the


physician's office for a scheduled health
maintenance examination. His father died
of a myocardial infarction at age 55 years.
Physical examination shows a tendon
xanthoma on the elbow. His serum total
cholesterol concentration is 360 mg/dL. A
mutation is most likely to be found in
which of the following genes?
(A)
(B)
(C)
(D)
(E)

A full-term 2-week-old male newborn has


cyanosis. Pregnancy and delivery were
uncomplicated. His lungs are clear, and a
midsystolic murmur is heard that is
loudest in the left third intercostal space
and associated with a thrill. Which of the
following is the most likely diagnosis?

apoA2
apoC2
apoE-4
LDL receptor
VLDL receptor

39

132.

A 47-year-old woman comes to the emergency department because of a 2-week history of intermittent
abdominal pain, nausea, and vomiting. She has had similar episodes sporadically during the past 4
years. Physical examination shows dehydration, jaundice, and upper abdominal distention. Laboratory
studies show hyperbilirubinemia. A CT scan and upper gastrointestinal series of the abdomen with oral
contrast are shown; the arrows indicate the abnormality. Which of the following is the most likely
cause of these findings?
(A)
(B)
(C)
(D)
(E)

133.

Annular pancreas
Cirrhosis of the liver
Duodenal constriction by the portal vein
Duodenal constriction by the superior mesenteric artery
Pyloric stenosis

A 37-year-old woman with right lower


extremity edema is evaluated because of
the sudden onset of shortness of breath
and pleuritic chest pain. A diagnosis of
pulmonary embolism is made. Which of
the following signs, if present on physical
examination, would be the most specific
indicator
of
pulmonary
arterial
hypertension in this patient?

134.

A 56-year-old man comes to the


emergency department because of a 4-day
history of colicky right flank pain that
radiates to the groin and hematuria.
Ultrasound examination of the kidneys
shows right-sided hydronephrosis and a
dilated ureter. Which of the following is
most likely to be found on urinalysis?
(A)
(B)
(C)
(D)
(E)

(A) Increased jugular venous


pressure
(B) P2 louder than A2
(C) Peripheral edema
(D) Presence of an S3
(E) Pulmonary crackles

40

Erythrocyte casts
Glucose
Leukocyte casts
Oval fat bodies
Uric acid crystals

135.

A 62-year-old man comes to the physician for a follow-up examination after he was diagnosed with chronic
inflammatory interstitial pneumonitis. Following pulmonary function testing, a biopsy specimen of the affected
area of the lungs is obtained. Compared with a healthy man, analysis of this patient's biopsy specimen is most
likely to show which of the following patterns of changes in the cell populations of alveoli?

(A)
(B)
(C)
(D)
(E)
(F)
(G)
(H)

136.

Type II Pneumocytes

Fibroblasts

A 63-year-old man with a 5-year history of congestive heart failure comes to the emergency department because
of a 1-month history of fatigue and labored breathing. Evaluation shows pulmonary edema. Furosemide is
administered. Which of the following sets of physiologic changes is most likely following administration of the
drug?

(A)
(B)
(C)
(D)
(E)
(F)
(G)
(H)

137.

Type I Pneumocytes

Na+K+Cl Transport in the


Thick Ascending Loop of Henle

Osmolarity of the
Medullary Interstitium

A 55-year-old man with a history of drug and


alcohol abuse undergoes operative placement of
a portosystemic shunt to relieve portal
hypertension. During this procedure, it is most
appropriate for the physician to anastomose a
major tributary of the portal vein to which of the
following vessels?
(A)
(B)
(C)
(D)
(E)

138.

A 52-year-old woman is admitted to the hospital


because of breast cancer metastatic to the liver.
Her prognosis is poor. She begs her husband to
stay with her at the hospital because she is afraid
to be left alone. Which of the following defense
mechanisms best explains her behavior?
(A)
(B)
(C)
(D)
(E)

Left gastric vein


Left renal vein
Splenic vein
Superior mesenteric vein
Umbilical vein

41

Water Absorption in the


Descending Loop of Henle

Denial
Displacement
Regression
Repression
Sublimation

Answer Form for USMLE Step 1 Sample Test Questions


Block 1 (Questions 1-46)
1.
2.
3.
4.
5.
6.
7.
8.
9.
10.

___
___
___
___
___
___
___
___
___
___

11.
12.
13.
14.
15.
16.
17.
18.
19.
20.

___
___
___
___
___
___
___
___
___
___

21.
22.
23.
24.
25.
26.
27.
28.
29.
30.

___
___
___
___
___
___
___
___
___
___

31.
32.
33.
34.
35.
36.
37.
38.
39.
40.

___
___
___
___
___
___
___
___
___
___

41.
42.
43.
44.
45.
46.

___
___
___
___
___
___

___
___
___
___
___
___
___
___
___
___

87.
88.
89.
90.
91.
92.

___
___
___
___
___
___

___
___
___
___
___
___
___
___
___
___

133.
134.
135.
136.
137.
138.

___
___
___
___
___
___

Block 2 (Questions 47-92)

47.
48.
49.
50.
51.
52.
53.
54.
55.
56.

___
___
___
___
___
___
___
___
___
___

57.
58.
59.
60.
61.
62.
63.
64.
65.
66.

___
___
___
___
___
___
___
___
___
___

67.
68.
69.
70.
71.
72.
73.
74.
75.
76.

___
___
___
___
___
___
___
___
___
___

77.
78.
79.
80.
81.
82.
83.
84.
85.
86.

Block 3 (Questions 93-138)

93.
94.
95.
96.
97.
98.
99.
100.
101.
102.

___
___
___
___
___
___
___
___
___
___

103.
104.
105.
106.
107.
108.
109.
110.
111.
112.

___
___
___
___
___
___
___
___
___
___

113.
114.
115.
116.
117.
118.
119.
120.
121.
122.

___
___
___
___
___
___
___
___
___
___

123.
124.
125.
126.
127.
128.
129.
130.
131.
132.

42

Answer Key for USMLE Step 1 Sample Test Questions


Block 1 (Questions 1-46)

1. C
2. E
3. B
4. E
5. B
6. F
7. D
8. B
9. B
10. A
11. A
12. A

13. A
14. B
15. A
16. A
17. A
18. A
19. E
20. B
21. A
22. D
23. E
24. C

25. C
26. B
27. C
28. A
29. D
30. C
31. B
32. E
33. D
34. C
35. E
36. B

37. B
38. A
39. A
40. A
41. B
42. C
43. F
44. A
45. C
46. D

Block 2 (Questions 47-92)

47. C
48. C
49. B
50. B
51. C
52. C
53. E
54. D
55. A
56. D
57. A
58. D

59. G
60. D
61. D
62. C
63. C
64. C
65. C
66. C
67. A
68. D
69. B
70. D

71. B
72. C
73. D
74. D
75. D
76. C
77. G
78. E
79. E
80. D
81. C
82. D

83. A
84. B
85. D
86. A
87. C
88. B
89. B
90. E
91. F
92. D

Block 3 (Questions 93-138)


93. E
94. D
95. C
96. A
97. E
98. A
99. A
100. B
101. E
102. E
103. E
104. E

105. A
106. C
107. E
108. E
109. A
110. A
111. B
112. C
113. E
114. A
115. A
116. B

117. A
118. A
119. E
120. C
121. B
122. A
123. E
124. A
125. E
126. A
127. E
128. B

43

129. D
130. E
131. E
132. A
133. B
134. E
135. E
136. H
137. B
138. C

FPCFromClipboardUntitled
Block 1
C Acute abdominal pain in a fertile woman is very frequently going to be an
ectopic question. Associated fun fact: the most common cause of secondary amenorrhea
is pregnancy.
E Parents can be overly protective of their children with chronic diseases
(diabetes, lupus, etc). This child doesnt have SCID, so going to school isnt going
to kill them. The child is an overall reasonable health with only mild symptoms.
Vulnerable child syndrome is characterized by unreasonable parental anxiety. *
B HGPRT = high uric acid. Self-mutilating behavior (e.g. bad finger biting) is one
of the more specific clinical features.
E Functional parathyroid adenomas can cause elevated parathyroid hormone (PTH),
which results in hypercalcemia and hypophosphatemia. Hypercalcemia is characterized
by the rhyming symptoms of: stones (renal, biliary), bones (including bone pain to
osteitis fibrosa cystica), groans (abdominal pain, n/v), thrones (polyuria,
constipation), and psychiatric overtones (from depression to coma). *
B You are seeing a tube inside of a tube in the pathology specimen. Yes, look
again: bowel within bowel. Intussusception. Currant jelly stools. Classic history
(sudden onset colicky pain, palpable mass). Now, a right lower quadrant lesion that
causes pain and hematochezia, particularly in a child? Then you would get to think
of Meckels (with its rule of 2s).
F While staph aureus is the most common cause of hematogenous osteomyelitis in
children, Salmonella is the most frequent cause for patients with sickle cell anemia
(a test favorite). I think I may have even had this fact tested twice on my Step.
D Reyes syndrome is the reason why children dont receive aspirin (except to
treat Kawasakis disease, of course). Reyes syndrome is characterized by vomiting
followed liver damage (hepatic steatosis and hepatomegaly) and encephalopathy
beginning with irritability and aggressive behavior and potentially progressing to
coma.*
B Bisphosphonates work by decreasing osteoclast activity (thereby reducing bone
resorption). Choice is F is the opposite of how estrogen therapy works (RANKL is
found on osteoblasts, and its activation triggers osteoclasts and stimulates bone
resorption).*
B Azotemia, hemoconcentration, and hypotension/tachycardia are all results of
dehydration. This may lead to acute renal failure if allowed to continue (but this
diagnosis requires a change in creatinine of 0.3).
A Phenylephrine is an alpha-agonist nasal spray and oral medication that often
shows up on exams. Oxymetazoline (Afrin) is a similar alpha-agonist nasal spray.
Pseudoephedrine is another decongestant with a similar MOA thats less common now
that its a federally monitored component of methamphetamine production. Topical
alpha agonist decongestants are physically addictive and can cause miserable rebound
congestion (rhinitis medicamentosa). Systemic formulations can cause hypertension
and worsen prostate problems by causing prostatic smooth muscle constriction. Just
remember that tamsolusin (Flomax) is an alpha-5a antagonist, which relaxes prostate
muscle.
A She has nephrogenic diabetes insipidus (large volume dilute urine production
that cannot be reversed with the administration of exogenous vasopressin).
Aquaporins are the water channels that allow for the reabsorption of free water from
Page 1

FPCFromClipboardUntitled
the collecting ducts and the production of concentrated urine.
A The p-value corresponds to the likelihood of a type I error (a false positive).
A lower p-value means a lower acceptable likelihood of obtaining the same results by
chance, and thus, significant results can be reported more confidently (a 1% false
positive rate instead of a 5% rate).*
A Gonorrhea can change its pillus, which is responsible for adhesion to host cells
and the main antigen to which the host mounts an immune response. Neisseria
gonorrhoeae is able switch out different pilin genes, and for this reason, prior
infection does not confer long lasting immunity.*
B Crossed findings (upper motor neuron on one side, lower motor neuron on the
other) means a brainstem lesion. Left (ipsilateral) tongue, right-sided
(contralateral) weakness means the exiting left hypoglossal nerve has been affected
(within the left medulla). B is in the pyramid where the corticospinal tract runs to
control muscles (prior to the decussation).
A p53 is the quintessential tumor suppressor (it activate apoptosis).
Carcinogenesis of HPV is caused by insertion of the virus into the host DNA and
producing a protein which binds with an essential p53 substrate, functionally
inactivating the p53 and its apoptotic cascade. C (transactivation/TAX) is how HIV
and HTLV cause cancer. E (cmyc translocation) causes Burkitt lymphoma.*
A Atypical antipsychotics (e.g. clozapine, quetiapine, etc) are more likely to
improve negative symptoms (affective flattening, anhedonia, avolition, aphasia) when
compared to typical antipsychotics (e.g. haloperidol). They are no more effective at
treating positive symptoms (hallucinations, delusions). Highly testable fact.*
A Its the antibodies to surface antigens that are protective. Hemagglutinin is a
surface antigen that is responsible for clumping RBCs in vitro.
A All the vesicles contain the same viral infection, so all should have the same
appearance on gel. Choice D is what a gel looks like without using a restriction
enzyme digest (no discrete bands of specific weights).
E Xanthogranulomatous pyelonephritis (XGP) is a rare form of chronic
pyelonephritis, especially associated with Proteus infection. Tumor-like growth,
upper urinary tract infection, and (this is key) lipid-laden foamy macrophages make
this neither acute pyelo nor cancer. Malacoplakia causes GU papules/ulcers,
typically of the bladder (not super important).
B Post-streptococcal glomerulonephritis is caused by the deposition of circulating
immune complexes (a type III hypersensitivity and a test-favorite).*
A As always, its almost better to ignore the pictures when possible. This
gentleman has a peptic ulcer, which we know is caused predominately by H. pylori
infection. H. pylori produces proteases and particularly urease, which allow it to
decrease the pH of the its local environment by cleaving urea into ammonia, which is
toxic to gastric mucosa. The picture demonstrates H pylori, which are evident with
silver staining.*
D Schistosomiasis is a parasitic worm particularly endemic in Africa (Egypt in
particular comes up a lot on questions) that is most associated with chronic
cystitis. Calcifications of the bladder wall are essentially pathognomonic. Chronic
infection is associated with an increased risk of squamous cell carcinoma of the
bladder (as opposed to the usual urothelial/transitional cell).*
E This patient has chronic kidney disease, as indicated by elevated serum
creatinine/BUN and evidence of anemia of chronic disease (normochromic normocytic).
Page 2

FPCFromClipboardUntitled
Poorly functioning kidneys do not hydroxylate 25-Dihydroxycholecalciferol to
1,25-Dihydroxycholecalciferol well nor produce adequate erythropoietin (hence the
CKD-related anemia). Patients with CKD development thus develop secondary
hyperparathyroidism due derangements in phosphate excretion and inadequate Vitamin D
activation resulting in hypocalcemia. Thus, we should expect to see low calcium,
high phosphorus, low 1,25 vitamin D, and low Epo, which is E.*
C Isoniazid can cause peripheral neuropathy due to its depletion of pyridoxine
(vitamin B6). B6 supplementation is therefore preventative. INH can also cause
drug-induced lupus (another test favorite) and sometimes severe liver disease (also
fair game).*
C The meningitis diagnosis is a giveaway. Your job is to know two things: 1) the
common pathologic organisms in different age groups or 2) that Neisseria is a gram
negative diplococcus.
B To amplify tiny fragments of DNA in order to detect their presence, we use PCR.
The question is a description of the process. Southern Blots are used to detect a
specific DNA sequence within a DNA sample.*
C Hookworm infection can cause intestinal irritation leading to chronic GI blood
loss and iron deficiency anemia. An otherwise healthy boy who lives in rural
Mississippi, probably playing in the mud in his bare feet like a hillbilly? Thats
the kind of kid who would get hookworm. Treatment is mebendazole (albendazole would
also be fine. These are your go to choices for anti-helminthics).*
A Androgen insensitivity is caused by a defective androgen receptor. DHT is
responsible for creating male genitalia during fetal sexual development. The default
human gender is female. So a genetically male patient with complete androgen
insensitivity is externally phenotypically female. Lack of response to adrenal
androgens prevents hair formation during puberty (adrenarche).*
D You need to memorize the list of drugs that induce and inhibit CYP450. Warfarin
has oodles and oodles of drug reactions for this reason with potentially dire
consequences.
C Leydig cells make testosterone. Leydig cell tumors arent always physiological
active, but those that are can cause masculinization. Granulosa cell tumors, on the
other hand, sometimes produce estrogen. Teratomas are oddballs that typically have
fat, hair, teeth, etc. Thecomas will not be on your test. Ovarian carcinoid is
highly unlikely to show up on your test, but if it did, it would likely present with
a classic carcinoid syndrome.*
B This patient has cystic fibrosis. The combination of respiratory and GI issues
is classic and caused by ineffective chloride transport and consequently thick
exocrine secretions, which clog up the airways and the pancreatic ducts.
E Polycythemia Vera is the red blood cell cancer. Symptoms are related to
hyperviscosity of the increased hematocrit. A classic symptom of polycythemia vera
is pruritus, typically with exposure to hot water (e.g. the shower). Super high
hematocrits are indicative. Low EPO means that the body is responding appropriate by
trying to tone down RBC production (which continues autonomously in PCV). JAK2
mutation, test favorite. The classic treatment is bloodletting, like they used to do
for everything back in the middle ages.*
D Sensitivity rules things out. Its TP / (TP + FN). So in order to calculate the
sensitivity of this test, we need the true positives (the 90 with cancer) and the
false negatives: the patients for whom the test is negative but actually do have
Page 3

FPCFromClipboardUntitled
prostate cancer. Thats D.*
C Polyarteritis nodosa is that only one that makes sense in explaining the diffuse
constellation of symptoms in this question. It affects small and medium arteries,
including those involving the skin, heart, kidneys, GI tract, etc. On imaging, a
rosary sign of small aneurysms strung like beads on a rosary is sometimes
mentioned. The biopsy shows an inflamed artery (arteritis). Angiodysplasia typically
causes painless GI bleeding (like that seen in diverticulosis). Kawasaki disease is
in children (5 days of fever, strawberry tongue, etc). Takayasu arteritis involves
big arteries, like the aorta, carotids, and subclavians (with classically diminished
upper extremities pulses, hence pulseless disease). Thromboangiitis obliterans is
also known as Buergers disease, a vasculitis seen essentially exclusively in
smokers that causes severe peripheral arterial disease (legs much more so than
hands). Wegeners is typified by the combination of renal and lung (and sinus)
findings in conjunction. The picture would should granulomas, and C-ANCA would
likely be positive, because its a board exam.*
E Androgens stimulate sebaceous glands and cause acne. In girls, this is primarily
due to adrenarche (DHEA/DHEAS androgen production made by the adrenal gland the zona
reticularis). Boys can also blame testosterone from gonadal puberty (pubarche).*
B Six-year-olds typically understand the finality of death. Infants have no
understanding, whereas preschool age children often think of deaths in reversible or
metaphorical terms.
B Thiazides (typically used an antihypertensives) also increase calcium resorption
in the distal tubule and are therefore useful in preventing calcium oxalate stone
formation in patients with hypercalciuria (the mechanism is not really worth
learning). Thiazides block the Na-Cl symporter, as opposed to loop diuretics, which
block the triporter, and acetazolamide, which blocks carbonic anhydrase in the
proximal tubule.*
A Pyknosis and nuclear fragmentation are part of the process of apoptosis.
A Acetaminophen (Tylenol) can cause fulminant hepatic failure in overdose. This
will probably be on your test.
A The infraspinatus and teres minor are responsible for external rotation. Both
the infraspinatus and supraspinatus muscles are innervated by a suprascapular
nerve.*
B Osteogenesis imperfecta (blue sclera, lots of fractures [they even occur
prenatally]) is a defect in type 1 collagen.
C Filgrastim is a granulocyte colony stimulating factor, which are drugs use to
increase white blood cell count in patients with leukopenia. Leucovorin (folinic
acid) sounds like it would also be right; its used to prevent bone marrow
suppression in patients taking methotrexate. Darbepoetin (like erythropoietin) is
used to stimulate red blood cell production.*
F A Nystatin mouthwash is the treatment of choice for oral candidiasis. Its
topical, its easy, it works. For tongue thrush, the patient can spit it out (and
therefore no chance for side effects). For pharyngeal/esophageal candidiasis, the
patient can swish and swallow. Inhaled cortisteroids, particularly if used without
an air chamber (spacer) can cause local immune suppression when contacting the oral
mucosa, leading to thrush.*
A Turner syndrome (you may remember lymphedema of the neck by another name: cystic
hygroma). The 45,X gives it away though anyway. Mosaic Turners syndrome and the
Page 4

FPCFromClipboardUntitled
mosaic trisomies result from nondisjunction during mitosis. In total
monosomy/trisomies, the cause is nondisjunction during meiosis. Uniparental disomy
is essentially only tested via the Prader-Willi and Angelman syndromes (chromosome
15).
C GVHD sucks. Skin and GI lesions are especially common sites (mucosal tissues are
rapidly dividing and thus prone to attack).
D The baroreceptors are stretch receptors (the more fluid in the vessel, the more
they fire). So a patient with hemorrhagic shock will see a decrease in the
baroreceptor firing rate. Activation of RAAS will result in increased vascular
resistance (vasoconstriction) in order to maintain blood pressure. And capillaries,
such as those in the kidney, will be primed for resorption and not filtration (no
one wants to pee out good dilute urine when theyre dehydrated). Likewise, systemic
capillaries will prefer to hold onto plasma and not let it leak into the
interstitium (third-spacing).*
Block 2
C Home canned/bottled food is a buzz-term for botulism. Thats also why the
bottles you buy in the store have that little pop up lid that stick out when you
open it. If theres botulism forming air in the bottle, then the lid pops up (and
you shouldnt eat it!). Botulinum toxin inhibits the release of acetylcholine (which
actives the nicotinic receptors necessary for skeletal muscle contraction).*
C Blood flow also increases during exercise. The more anaerobic metabolism you use
and lactate build-up you have, the more hyperemia you need to clear out the waste
products.
B Aminoglycoside antibiotics (gentamycin, amikacin, etc) are powerful antibiotics
especially useful for bad gram negative infections. Bad side effects are permanent
hearing loss (ototoxicity) and renal failure. Both are important to know. Torsades
de Pointes (choice E) can be caused by fluoroquinolones (e.g. cipro), as these drugs
prolong the QT-interval.
B Memorize aspirins acid-base effects: metabolic acidosis and respiratory
alkalosis. Note, this is actual respiratory alkalosis, not simply normal respiratory
compensation for metabolic acidosis.
C This patient has hepatitis (elevated liver enzymes) due to active Hepatitis C
infection. Hep C and HIV infection are both associated with intravenous drug use.
While most patients with Hep A will clear the virus after their acute illness, Hep C
causes chronic infection in 80% of patients, which may lead to cirrhosis over time
(~20 years).*
C Osgood-Schlatter is also known as apophysitis of the tibial tubercle. Its due
to chronic stress/irritation at the insertion of the patellar ligament on the tibial
tubercle. Its classically seen in the teenagers doing repetitive vigorous activity
(running, jumping). The radiograph demonstrates classic fragmentation of the tibial
tubercle (which isnt necessary to know to get the question correct).*
E Gram positive rods in a diabetic foot wound (or a World War I soldier fighting
in a trench) means Clostridium perfringens (the causative organism of gas gangrene).
Crepitance means gas in the tissues, which is produced as a byproduct of its highly
virulent alpha toxin.*
D Those are varicose veins, and theyve described symptomatic varicosities.
Incompetent valves allow reflux of blood into the dependent feet and legs. The
Page 5

FPCFromClipboardUntitled
pooling blood increases hydrostatic pressure, causing edema.
A Electrical alternans on boards means a big pericardial effusion (and usually
cardiac tamponade). The heart cannot fill properly, preload decreases, hypotension
and tachycardia ensue, fluid backup leads to elevated JVP.*
D Antibiotic-associated diarrhea caused by clostridium difficile can be tenacious,
difficult to treat, and even fatal. Alcohol-based rubs are not sufficient to kill
the spores. Handwashing with soap is necessary, and equipment should be autoclaved
to clean it.
A Ballet dancers (wrestlers, models, ballet dancers, athletes who need to make
weight, and particularly young women in general) all have eating disorders on Step
1. Folic acid is involved in the production of both red and white blood cells.
Remember, low folate leads to macrocytic anemia and hypersegmented neutrophils. B12
deficiency (not an answer choice) leads to macrocytic anemia and neurological
changes (including SCID in severe cases).
D ITP causes immune-mediated consumption of platelets, hence the low platelet
count and petechiae. The bone marrow biopsy results demonstrate that the body has
appropriately increased platelet production, meaning that this is not a platelet
production issue. While TTP has a similar acronym, its an entirely different
disease with a classic pentad: thrombocytopenia (low platelet count),
microangiopathic hemolytic anemia, altered mental status, renal failure, and fever.
G Vincristine (a mitosis inhibitor) frequently causes peripheral neuropathy, which
can be severe and irreversible. Other fun associations are Bleomycin with pulmonary
fibrosis, Cyclophosphamide and bladder cancer, and Doxorubicin with dilated
cardiomyopathy.*
D The arrowed fluid is contained in a space behind the stomach but in front of the
retroperitoneal structures (e.g. the pancreas), i.e. the lesser sac.*
D People in their 60s dont spontaneously become schizophrenic with any frequency
for that to be ever be the correct answer. Likewise, Alzheimers is a slowly
progressive cognitive decline (dementia), not an acute decline in mental status
(i.e. delirium). This patient is delirious. Common causes in the elderly includes
medications, infections, and being in the hospital (particularly the ICU).*
C Anaphylaxis is treated with epinephrine.
C Logic would dictate that a fracture of the inferior orbital wall might affect
the infraorbital artery, and logic would be right.*
C If you know you are getting a drug, then you are not blinded: its an open label
trial. It is a clinical trial though.*
C Ah, you really want to pick A for cat scratch fever. But sore throat,
adenopathy, and a positive Heterophile antibody test means mononucleosis (i.e.
Mono), caused by Epstein-Barr Virus. The heterophile antibody test is due to a cross
reaction with horse or sheep red blood cells, which are agglutinated in vitro by the
antibodies in the patients serum.*
C RSV, like all respiratory viruses, spreads via respiratory droplet. Babies are
too young to wheeze because of asthma; they wheeze because of RSV.
A Malonyl-CoA inhibits the rate-limiting step in the beta oxidation of fatty acid.
Logically, resting muscle requires less energy (and thus less need for fatty acid
breakdown) than active muscle.*
D Recurrent respiratory infections could be a lot of things. But then they mention
the dextrocardia. Kartagener syndrome is the combination of situs inversus and
Page 6

FPCFromClipboardUntitled
defective cilia (due to a mutation in dynein), where the inability to effectively
clear secretions results in recurrent sinusitis and bronchiectasis.
B Choriocarcinoma is a much-feared complication of a molar pregnancy. It is a
cancer of the bHCG-producing syncytiotrophoblasts found in the placenta.
D The suprachiasmatic nucleus of the hypothalamus controls circadian rhythms. A
few more key thalamic nuclei are worth knowing: Supraoptic releases vasopressin
(ADH). The lateral nucleus controls thirst and hunger. The ventromedial controls
satiety. Anterior controls temperature. The paraventricular nucleus releases CRH,
TRH, and oxytocin.
B This patient has symptomatic anemia. Its microcytic nature implies iron
deficiency, which is most commonly due to occult blood loss. In the elderly, the
concern is colon cancer. In a reproductive age female, iron deficiency is more
commonly secondary to uterine pathology.
C Factor V leiden is by far the most common heritable cause of hypercoagulability.
Keep in mind that many most likely questions are actually asking you for the most
common cause.*
D This is rheumatic fever from group A strep pharyngitis. This is thankfully rare
now, as we routinely treat Step throat with antibiotics. The cause of all the damage
is due to cross-reactivity of Strep antigens with the tissues of the heart, joints,
skin, and brain. Anti-streptolysin O (ASO) and anti-DNase titers will be high.
D The Odds Ratio (OR), if you dont simply have it memorized, is computed exactly
as you would guess. Its the odds of you getting a disease with the treatment (or
risk factor) over the odds of you getting a disease without the treatment. In this
case: 100/200 divided by 300/300 = 1/2.*
D Rickets. Pectus carinatum is also known as pigeon chest (protruding sternum)
and bead-like enlargement of the costochondral junctions is describing a rachitic
rosary. Rickets is caused by vitamin D deficiency (either dietary or functional).
Osteoblasts in patients with rickets lay down excess unmineralized osteoid, as they
are less able to mineralize osteoid into mature bone without sufficient vitamin D.*
C Lymphatic spread of disease moves through lymphatic channels from distal to
proximal. The medial side drains to the superficial inguinal nodes. Much of the
lateral side will stop at the popliteal nodes prior to ascending the thigh.
G Sulfonylurea medications (glipizide, glyburide) stimulate the pancreas to
secrete more insulin. For this reason, they are most efficacious early in the
disease process when pancreas still has remaining functional reserve.
E Those are sickle cells on the smear. LUQ pain on test questions almost always
means splenic pathology. All sickle cell kids will eventually infarct their spleen.
E Subacute combined degeneration (progressive peripheral sensory and motor loss)
is a late sign of B12 deficiency, which is common in old people. On board exams, a
geriatric patient who lives alone and may have a tea and toast diet is likely to
have vitamin deficiencies, particularly of folate and B12.
D An OR greater than 1 signifies increases odds/risk/likelihood. If the 95% CI
range does not include 1, then the difference is statistically significant (though
not necessarily clinically meaningful).*
C Common sense is key, particularly for counseling-type questions. Patients have
autonomy and can do whatever they want; its your job to explain the risks and
benefits. The patients ultimately make their own treatment choices.
D She is taking anabolic (androgenic) steroids as a performance enhancing drug.
Page 7

FPCFromClipboardUntitled
Being an athlete on Step 1 is never a good thing.*
A A new blistering disease in an older person is typically going to be a pemphigus
question. Then you just have to remember the difference between bullous pemphigoid
vs pemphigus vulgaris. Bullous pemphigoid is characterized be the loss of
hemidesmosomes that bind keratinocytes to the basement membrane, resulting in bulla
(big blisters) in areas of friction, choice A. Patients with pemphigus vulgaris lose
their desmosomes (which bind keratinocytes to each other), so that their skin is
super friable, which results in ulceration. Mouth ulcers are more common in PV.*
B This is a (prospective) case series. There is no control (and certainly no
blinding).*
D Pregnant patients should avoid fish and seafood products that are high in
mercury. In general, this means that shellfish and big salt-water fish should be
avoided (tuna, swordfish, shark, king mackerel, tilefish, etc)*
A Air and fluid = hydropneumothorax. If that fluid is blood (s/p stabbing), its a
hemopneumothorax. Lack of mediastinal shift indicates that its not under tension.*
C Middle-aged woman with progressive shortness of breath? Think of idiopathic
pulmonary fibrosis, a restrictive lung disease.
B Finasteride (aka Propecia) is used for male pattern baldness and prostate
hypertrophy. Its a 5-alpha-reductase inhibitor, which prevents the conversion of
testosterone to dihydrotestosterone (DHT).
B Gout, gout, gout. Allopurinol helps prevent flares but does nothing to treat
them. Treatments of choice for an acute flare are NSAIDS or colchicine.
E Splitting is an immature defense mechanism often employed by patients with
borderline personality disorder. When splitting, a person fails to see others as
capable of having both positive and negative qualities; at any given time, its all
or nothing.*
F Prolonged bleeding time with normal clotting factors (as evidenced by normal
PT/INR and PTT) and a sufficient platelet count is going to be von Willebrand
disease, the most common hereditary coagulation abnormality, which by either
deficiency or mutation results in dysfunctional platelets. A temporary treatment for
uncontrolled bleeding is vasopressin, which causes for addition vWf release. Factor
VIII concentrate is a more dramatic and more effective treatment, as it also
contains vWf.*
D Diffuse low-level ST elevation means pericarditis. These patients often complain
of pleuritic chest pain, which is somewhat alleviated by sitting up and leaning
forward, and have distant heart sounds. Common test causes include viruses, uremia,
and 2-3 weeks after myocardial infarction (Dressler syndrome). *
Block 3
E Profuse super-watery diarrhea means cholera (the so-called rice water stool).
That said, regardless of the cause, you treat all causes of volume loss with volume
replacement (normal saline)!
D Torticollis is a type of focal dystonia, which is a type of EPS (extrapyramidal
symptoms) caused by neuroleptics (antipsychotics), such as haloperidol.
Uncontrollable facial grimacing is a description of tardive dyskinesia, a rarer
sometimes permanent EPS more associated with long-term antipsychotic use.*
C Thats a litany of symptoms with only one reasonable single possible cause out
of the provided choices: hyperthyroidism. Elevated thyroid hormone can manifest as
Page 8

FPCFromClipboardUntitled
anxiety, GI hypermotility, tachycardia and a-fib, weight loss, heat intolerance,
etc. Thyroid disorders are very high yield. Pheochromocytomas (choice E) can cause
some of the same symptoms in an episodic fashion (more typically panic attacks,
episodic severe hypertension, headache).
A Narcotic use for acutely painful conditions is both reasonable and important.
Short-term use (immediately post-surgical) does not lead to long term dependence.
And yes, drugs addicts should also receive narcotics to control pain.*
E The most important cause of papillary necrosis is analgesia [abuse] nephropathy,
a type of kidney damage caused by long-term use of OTC pain medications. Ischemia
from disruption of blood flow from the vasa recta causes necrosis and sloughing of
the renal papilla. Hematuria and sterile pyuria are common.*
A Altitude sickness can cause high-altitude pulmonary edema (a type of ARDS). The
x-ray demonstrates diffuse bilateral fluffy infiltrates.*
A The genetic questions always seem to hinge on remembering (or deducing) that T
in DNA is replaced by U in RNA.*
B If you think about this logically, what we have here is a congenital intolerance
to breastmilk: galactosemia, in which the body cannot convert galactose to glucose
(resulting in an accumulation of Galactose 1-phosphate). They then list the findings
and tests used to diagnose it. Lactose (the disaccharide in milk) is composed of
glucose + galactose.*
E Endothelial tight junctions permeability is increased in response to injury and
inflammation, allowing migration of white blood cells and friends to the site of
injury.*
E Ventricular fibrillation is the most common cause of sudden cardiac death
immediately after myocardial infarction. This is why we have AEDs all over the place
now. Papillary muscle rupture classically occurs 2-7 days after an MI and results in
massive life-threatening mitral regurgitation. Free ventricular wall rupture after
an MI can result in cardiac tamponade. *
E PPIs raise pH by preventing the normal secretion of HCl into the stomach by
parietal cells. The body attempts to counteract this unnaturally basic gastric pH
with compensatory hypertrophy. If our medications are moving a measured element
beyond its set-point, the body nearly always tries to compensate.*
E The whole afferent/efferent thing is worth knowing. When the efferent arteriole
is independently constricted, the blood can get into the glomerulus but has
difficulty getting out. So more blood spends a greater amount of time in the
glomerulus being filtered: GFR up, filtration fraction up, but overall blood flow is
decreased due to the increased resistance of the system as a whole.
A Benzodiazepines (including alprazolam aka Xanax) are effective at treating
anxiety conditions. Ideally, their use should be limited to bridge therapy awaiting
the efficacy of safer less habit-forming maintenance medications like SSRIs and
buspirone.*
C If you dont eat enough calories to run your metabolism, your body will mobilize
its stores. We burn fat through fatty acid oxidation.
E Chronic microcytic anemia in a patient with normal iron studies should make you
think of thalassemia. -Thalassemia minor is the most common and is typically quite
mild. As there is decreased beta chain production, there is a relative excess of
alpha chains.*
E Tetracycline use during childhood is commonly associated with tooth
Page 9

FPCFromClipboardUntitled
discoloration, typically yellow-brown (sometimes described as gray).*
A Targetoid rash after a woodland excursion means lyme disease, caused by Borrelia
burgdorferi, carried by the Ixodes tick. Rash (erythema migrans), viral syndrome
symptoms, fatigue, and poly-arthritis are common. Lyme carditis typically manifests
as AV block.*
A Adalimumab is the only TNF inhibitor on the list. The fancy targeted therapies
are all monoclonal antibodies and thus end in ab.*
B The arrow is pointing to a neutrophil (multilobed nucleus). Main fighter of the
immune system in acute inflammation and bacterial infection (such as aspiration
pneumonia). C5a is a chemotactic factor for PMNs.*
C AIDS retinitis is caused by CMV (typically seen with CD4 count less than 50).
Treatment is with ganciclovir.*
E This patient has stress incontinence, a common complaint in women after vaginal
childbirth, the risk of which increases with number of deliveries, the size of the
baby, and use of forceps, etc. The pelvic floor muscles and urogenital diaphragm are
innervated primarily by the S3-4 nerve roots.*
A Chronic anovulation is a common cause of infertility. Long periods are often
anovulatory, where lack of an LH surge leads to unchecked estrogen and prevents
ovulation and the secretory and menstrual phases that follow, leading to chronic
proliferative-phase endometrium and irregular menses, which can be long or short,
often light (as only the endometrial tissue that outgrows its blood supply sloughs
off).*
A GBS comes from a mothers colonized vagina and is the most common cause of
neonatal sepsis. Women who are GBS+ should receive PCN prophylaxis prior to delivery
to prevent exposure to the fetus during delivery.*
B Psychogenic polydipsia (PPD) is associated with several psychiatric conditions,
particularly schizophrenia. It is also sometimes felt to be secondary to the
dry/cotton mouth seen with certain medications, including antipsychotics. Patients
present with hyponatremia due to their excessive free water intake. In some cases, a
water deprivation test is necessary to distinguish PPD from diabetes insipidus.*
A Foot drop after compression in the lower leg = common fibular/peroneal nerve.
Nerve compression syndrome = one reason why its important that casts not be too
tight.*
A The closer R is to 1, the more closely the data points should fit to a line. I
think the NBME is trying to imply that I will have poorly controlled diabetes in the
future.*
E VEGF is a major tissue growth factor activated by injury, cytokine release
(infection, inflammation) and hypoxia that promotes angiogenesis and also increases
vascular permeability (hence the edema). This increased permeability aids in the
movement of proteins and white blood cells to the site of injury.*
C In this question, they have described the components of congenital rubella. The
distributed purpura (a result of extramedullary hematopoiesis) is a description of
the classic blueberry muffin rash (which you are highly unlikely to hear actually
described as such on a test).*
B Fatigable weakness, particularly of the eyes, is classic myasthenia gravis, an
autoimmune condition cause by antibodies that block acetylcholine receptors at the
postsynaptic neuromuscular junction. The arrow is pointing to the patients
prominent thymus. Thymoma and thymic hyperplasia are both in common in the patients
Page 10

FPCFromClipboardUntitled
with MG, and thymectomy is curative in a portion of patients. Small cell carcinoma
of the lung is associated with Lambert-Eaton myasthenic syndrome, a disease caused
by autoantibodies against presynaptic calcium channels. These patients classically
begin weak and an experience a warm up effect after some use (they ultimately
become weak again though).*
A Renal artery stenosis causes decreased blood flow to the supplied kidney, which
results in the activation of the Renin-Angiotensin-Aldosterone System, because the
hypoperfused kidney secretes renin from the juxtaglomerular cells. The normal kidney
has normal renin secretion. However, the additional circulating aldosterone will
cause the normal kidney to retain sodium/water.*
E Tons of exercise followed by renal failure is always going to be rhabdomyolysis.
Dark urine (without actual blood in it) is due to myoglobinuria, which can cause a
false positive urine dipstick.
A In additional to thyroid problems, amiodarone can cause interstitial lung
disease, including potentially fatal non-reversible pulmonary fibrosis. Digoxin is
famous for its narrow therapeutic range and numerous side effects, including
green-yellow vision changes. Lisinopril can cause angioedema, as well as
hyperkalemia. Metoprolol can exacerbate asthma in addition to causing dizziness,
bradycardia, and hypotension. Procainamide can cause drug-induced lupus.*
E Delayed separation of the umbilical stump is a classic clue for leukocyte
adhesion deficiency. ICAM is the defective ligand most often asked about as a second
order question.
A This patient has Conns syndrome (primary hyperaldosteronism), most commonly
caused by a hyperfunctioning adrenal adenoma. This is an important and highly tested
cause of refractory hypertension. The patient has lab abnormalities consistent with
high aldosterone (high sodium, low potassium, metabolic alkalosis) with an
appropriately suppressed renin.*
E A chronic heaped up ulcerative lesion of the skin, especially on sun-exposed
areas in people who spend time outdoors, means cancer. If its keratinocytes at play
(and not melanocytes), that leaves you with either squamous cell or basal cell
carcinoma.
B Alcoholics (and any person with an altered level of consciousness) are a set-up
for aspiration pneumonia: classically RLL, classically foul-smelling. The organism
on tests will be Klebsiella.
D Cholesterol xanthomas, horrible serum cholesterol levels, and early death by MI
are all signs of familial hypercholesterolemia (caused by a defect in the LDL
receptor).
E All of the congenital heart defects and their associated murmurs are high-yield
and worth memorizing. Understanding murmur physiology is also high yield in general.
In this case, we have a cyanotic heart condition in a newborn. Tetralogy of Fallot
is the most common cyanotic heart lesion on tests and in real life. ASDsfixed split
S2are left-to-right (non-cyanotic lesions), at least until they reverse down the
line (Eisenmenger syndrome). Clinically significant PDAs are alluded to by their
continuous machinery murmur.
E This is what a correlation is. A negative r-value means that the relationship
between the variables is inverse (not direct), so as one goes up, the other goes
down.*
A An annular pancreas occurs when the pancreas is wrapped around the second
Page 11

FPCFromClipboardUntitled
(descending) portion of the duodenum. When symptomatic (in adults, typically when
there is superimposed pancreatitis), it can block flow of GI contents through the
intestines.*
B The left sided system is much higher pressure than the right side, hence the
aortic valve closing is usually louder than pulmonic valve. A P2 louder than A2
means that the pulmonary artery pressure is significantly elevated.*
E Calcium oxalate stones are the most common variety of kidney stones, but uric
acid stones make up 5-10% as well. None of the other choices are associated with
renal calculi of any variety.*
E The patients chronic inflammatory pneumonitis is killing off his lung
parenchyma (composed primarily of type I pneumocytes). Type II pneumocytes, in
addition to making surfactant, can replicate in order to replace type I pneumocytes,
so they will be increased. Chronic interstitial inflammation results in fibrosis,
hence an increase in fibroblasts.*
H Furosemide is the prototypical loop diuretic, which works by blocking the
triporter and preventing the reabsorption of 1 K, 1 Na, and 2 Cl ions. So less
K/Na/Cl ion transport causes decreased osmolarity of the medullary interstitium
(where these ions would normally enter). Water follows solute, so with less ions
reabsorbed, less water will be reabsorbed.
B Surgical portosystemic shunts are most often performed by anastomosing the
splenic vein to the nearby left renal vein. The splenic and SMV join to form the
main portal vein. are portal system veins. More commonly, a TIPS procedure is
performed to create an intrahepatic shunt between the portal and hepatic veins.*
C If an adult is taking on behaviors common to children, its called regression
(dont feel bad if its something you might do yourself).

Page 12

Das könnte Ihnen auch gefallen